0% found this document useful (0 votes)
107 views65 pages

1-3. Reading Materials - QAM - MT (Upto Matrix)

This document provides an overview of a Quantitative Analysis for Managers course offered by Chittagong University Center for Business Administration. The objective of the course is to build students' quantitative knowledge for business decision making. The course covers quantitative techniques like functions, matrices, calculus, data analysis, forecasting, linear programming, and other topics. It will be evaluated through midterm and final exams, class tests, and class participation. The course is taught by Shahed Bin Sadique and uses textbooks on management science and quantitative analysis.

Uploaded by

Rifat Hoque
Copyright
© © All Rights Reserved
We take content rights seriously. If you suspect this is your content, claim it here.
Available Formats
Download as PDF, TXT or read online on Scribd
0% found this document useful (0 votes)
107 views65 pages

1-3. Reading Materials - QAM - MT (Upto Matrix)

This document provides an overview of a Quantitative Analysis for Managers course offered by Chittagong University Center for Business Administration. The objective of the course is to build students' quantitative knowledge for business decision making. The course covers quantitative techniques like functions, matrices, calculus, data analysis, forecasting, linear programming, and other topics. It will be evaluated through midterm and final exams, class tests, and class participation. The course is taught by Shahed Bin Sadique and uses textbooks on management science and quantitative analysis.

Uploaded by

Rifat Hoque
Copyright
© © All Rights Reserved
We take content rights seriously. If you suspect this is your content, claim it here.
Available Formats
Download as PDF, TXT or read online on Scribd
You are on page 1/ 65

Chittagong University Center for Business Administration (CUCBA)

Faculty of Business Administration


Program: EMBA
Course Title: Quantitative Analysis for Managers, Course Codes: 613
Credit: 3:00 (Three)
Objective: The main objective of this course is to build-up the quantitative knowledge of
the students related to business decision making. At present business faces a
complex situation than ever before. The success of an organization basically
depends on proper decision making to clarify a complex situation and to make
a proper decision, the use of quantitative techniques is essential. In this regard
every business executive should have enough knowledge of quantitative
analysis for decision making. This course covers comprehensive
understanding of quantitative techniques related to business decision making.

Course Outline

Mid-Term Exam:
1. Introduction: Basic concepts of quantitative analysis, importance & limitations.
2. Function, Equation and Co-ordinate Geometry.
3. Matrix Algebra and its application in business.
4. Calculus and its application in business.
5. Data Collection & Analysis: Data and its type, collection of data, measures of
central tendency and dispersion.

Final Exam:
6. Forecasting Methods: Correlation and Regression
7. Linear Programming
8. Waiting Line Theory
9. Transportation Problem and Assignment Problem

Basic Texts:
1. Mathematics for Managerial Decisions - 2nd Edition
- Robert L. Childress, Robin D. Gorsky & Richard M. Witt.
2. Statistics for Management - 7th Edition
- Richard I. Levin & David S. Rubin
3. An Introduction to Management Science
Quantitative Approaches to Decision Making - 9th Edition
- David R. Anderson, Dennis J. Sweeney & Thomas A. Williams
4. Quantitative Analysis for Management
- Barry Render, Ralph M. Stair, Jr., Michael E. Hanna, T. N. Badri; Pearson

Semester Evaluation
1. Class Evaluation: Class Test - 10 % + 10%
2. Mid-Term Exam: - 40%
3. Final Exam: - 40%
Total: - 100%

1
Lecture Plan

Lecture - 1 : Introduction
Lecture - 2-5 : Function, Equation & Co-ordinate Geometry
Lecture - 6-11 : Matrix algebra and its application in Business
Lecture - 12-17 : Calculus and its application in Business
Lecture - 18-23 : Data Collection & Analysis
Lecture - 24 : Class Test & Presentation.

Lecture - 25-30 : Forecasting Method: Correlation & Regression


Lecture - 31-36 : Linear Programming
Lecture - 37 : Class Test
Lecture – 38- 39 : Assignment Problem
Lecture - 40-42 : Transportation Problem
Lecture - 40-42 : Waiting line theory
Lecture - 45 : Class Test & Review.

Course Teacher:
1. Shahed Bin Sadique
Department of Accounting & Information Systems
University of Chittagong.
Mobile: 01819-386100
E-mail: [email protected]

2
Quantitative Business Analysis: An Introduction
Introduction:
QAM, an approach to decision making based on the scientific method. It is a relatively
new discipline. Its contents and boundaries are not yet fixed. The scientific management
revolution of the early 1900s, initiated by F. W. Taylor, provided the foundation for the use of
quantitative methods in management. But modern Management Science research is generally
considered to have originated during the World War II period, when teams were formed to deal
with strategic and tactical problems faced by the military. Quantitative techniques are used to
substantiate the decision being taken. We make decisions in our everyday life without even
noticing them. Decision-making is one of the main activities of a manager. Decision-making can
be either qualitative or quantitative. In qualitative decision-making, intuition, and subjective
judgment are used. Past experience with similar problems is often an important factor in choosing
a qualitative approach, as is the complexity and importance of a problem. Managers tend to use a
qualitative approach to decision-making when (a) the problem is fairly simple (b) the problem is
familiar (c) the cost involved are not great (d) immediate decisions are needed. Conversely,
managers generally prefer to use a quantitative approach when one or more of the following
conditions exist: (a) the problem is complex (b) the problem is not familiar (c) the costs involved
are substantial (d) enough time is available to analyze the problem. Thus, in simple situations
decisions are taken simply by common sense, sound judgment and expertise without using
mathematics. But when the problem is complex, the problem is especially important (e.g. a great
deal of money is involved) and the manager desires a thorough analysis before attempting to make
decision, the problem is new and the manager has no previous experience from which to draw and
the problem is repetitive then the manager cannot develop a good solution without the support of
quantitative analysis. The search of a decision may also be done by trial and error but such a search
may be cumbersome and costly. Preparative calculations may avoid long and costly research.
Doing preparative calculations is the purpose of QAM. QAM does mathematical scoring of
consequences of a decision with the aim of optimizing the use of time, efforts and resources and
avoiding blunders.
The tools of QAM are not from any one discipline rather Mathematics, Statistics,
Economics, Engineering; Psychology etc. have contributed to the newer discipline of knowledge.
Today it has become a professional discipline that deals with the application of scientific methods
for decision-making and especially to the allocation of scarce resources.
Quantitative techniques basically are those statistical and operations research or
programming techniques which help in the decision making process especially concerning
business and industry. All these techniques require preliminary knowledge of certain topics in
mathematics. These techniques involve the introduction of the element of quantities i.e. they
involve the use of numbers, symbols and other mathematical expressions. The quantitative
techniques are essentially helpful supplement to judgment and intuition. These techniques
evaluate planning factors and alternatives as and when they arise rather than prescribe courses of
action. As such quantitative techniques may be defined as those techniques which provide the
decision maker with a systematic and powerful means of analysis and help based on quantitative
data, in exploring policies for achieving predetermined goals. These techniques are particularly
relevant to problems of complex business enterprises.
Statistical techniques are those techniques which are used in conducting the statistical
inquiry concerning a certain phenomenon. They include all the statistical methods beginning from
the collection of data till the task of interpretation of the collected data. More clearly, the methods
of collection of statistical data, the technique of classification and tabulation of the collected data,
the calculation of various statistical measures such as mean, standard deviation, co-efficient of
correlation etc the techniques of analysis and interpretation and finally the task of deriving
inferences and judgment their reliability are some of the important statistical techniques.
3
Programming techniques or what is generally described as operations research are the
model building techniques used by decision makers in modern times. They include wide variety
of techniques such as LP, Theory of Games, Simulation, Network Analysis, Queuing Theory and
many other similar techniques.
Quantitative approach is widely employed in business. Areas of application include
forecasting, capital budgeting, capacity planning, scheduling, inventory management, project
management and production scheduling.

History of QBA:
OR/MS/DS concept is derived from the fact that ‘unity is strength’. When there is a
calamity to the nation, citizens of all shades of opinion, join together to do their might to solve the
problem. In the Second World War, there was a natural calamity to Great Britain from German
Forces of Hitler. Allied forces were threatened on land, sea and air. Superior weapons and strategy
of Germans became a real threat to allied forces from German submarines, U-boats and aircraft.
Government appealed to the people and requested talents from all walks of life to join together
and find a solution to the problem to overcome the threatening situation. Scientists from all
relevant disciplines from all organizations joined together in different teams. Each team is given
a problem. These combined efforts produced fantastic results. This signaled to the birth of
OR/MS/DS as a separate discipline. When there is a problem where an immediate solution is
needed, available talents are pooled together. They put their heads together, come up with a
probable solution and put up the same for the consideration of management. This system of
solution of problem through OR/MS/DS is applicable in all fields like government department,
public and private sectors, business enterprises, defense and others. Today, almost every large
organization or corporation in affluent nations has staff for applying OR/MS/DS and in
government the use of OR/MS/DS has spread from military to widely varied departments at all
levels. This general acceptance of OR/MS/DS has come as managers have learned the advantage
of the scientific approach on which OR/MS/DS is based. Availability of faster and flexible
computing facilities and the number of qualified OR/MS/DS specialists enhanced the acceptance
and popularity of OR/MS/DS.

Why QAM:
1. The problem is complex and the manager cannot develop a good solution without the
aid of quantitative analysis.
2. The problem is especially important (i.e. a great deal of money is involved) and the
manager desires a thorough analysis before attempting to make a decision.
3. The problem is new and the manager has no previous experience from which to draw.
4. The problem is repetitive and the manager saves time and effort by relying on
quantitative procedures to make routine decision recommendations.

Role of Qualitative and Quantitative Analysis:

Structuring Problem

Define the Problem Identify the Problem Determine the Criteria


Role of QAM in Business and Industry:
We can use QAM for so many objectives. But we mention just a few below:

4
1. Enables the management to decide when to buy and how much to buy.
2. Aid to decision-making and improve its quality.
3. Identification of optimum solution that is most appropriate.
4. Integrating the system as a whole.
5. Improve objectivity of analysis and clarity of thought.
6. Minimize the cost and maximize the profit.
7. Improve productivity and efficiency.
8. Success in competitions and market leadership.
9. Renders great help in optimum resource allocation.

Scope of QAM:
The scope of QAM is not confined to any specific areas like defense services or industrial
world. The scope is quite wide. It is useful in every field of human activities where optimization
of resources is required. Main fields where QAM is extensively used are listed below:
1. National plans and budgets.
2. Defense services operations.
3. Government developments and public sector units.
4. Industrial establishment and private sector units.
5. R & D and engineering divisions.
6. Public works department and construction of projects.
7. Business management and competition.
8. Agriculture and Irrigation projects.
9. Education and training.
10. Transportation and communication.
11. Home management and personal budgeting.

QAM and Business Management:


Marketing:
(a) Analysis of marketing research information
(b) Statistical records for building and maintaining an extensive market
(c) Sales forecasting
Production:
(a) Production planning, control and analysis
(b) Evaluation of machine performance
(c) Quality control requirements
(d) Inventory control measures
Finance Accounting and Investment:
(a) Financial forecast, budget preparation
(b) Financial investment decisions
(c) Selection of securities
(d) Auditing function
(e) Credit policies, credit risk and delinquent accounts
Personnel:
(a) Labour turn over rate
(b) Employment trends
(c) Performance appraisal
(d) Wage rate and incentive plans

Economics:
(a) Measures of gross national product and input-output analysis
5
(b) Determination of business cycle, long-term growth and seasonal fluctuations
(c) Comparison of market prices, cost and profit of individual firms
(d) Analysis of population, land economics, and economic geography
(e) Operational studies of public utilities
(f) Formulation of appropriate economic policies and evaluation of their effect
Research and Development:
(a) Development of new product lines
(b) Optimal use of resources
(c) Evaluation of existing products
Natural Science:
(a) Diagnosis the disease based on data like temperature, pulse rate, blood pressure etc
(b) Judging the efficiency of a particular drug for curing a certain disease
(c) Study of plat life.

Methods of QBA:
1. Analytical method: The method of solution depends on classical steps and techniques in
Mathematics like use of differential calculus, integration, set theory, matrix and co-
ordinate geometry. Examples are EOQ and graphical solution for the product mix through
linear programming. This is a deterministic method.
2. Trial and error method: Some problems and models fail to yield a solution through
classical, mathematical or graphical methods. Trial and error method is used here. In this
method a certain algorithm is developed. One starting point is an initial solution, which is
the first approximation. The method of solution is repeated with a certain set of rules so
that initial solution is gradually modified at each subsequent solution till optimal solution
is reached. There are certain criteria laid down to check whether the solution has become
an optimal solution. The classical example of trial and error method is simplex method of
linear programming. This is a deterministic method.
3. Simulation method: Solution of problems using principles of statistics, sampling and
probability is called simulation method. This method is applied where the data is
insufficient or where the situation is quite uncertain or when it is impossible to generate
data by direct measurement. In such situations, samples are created as faithfully as possible
to represent the real situation called the ‘universe’. Random tables, mechanical devices
and electronic computers are used in order to establish random nature of events. Typical
example is Monte Carlo Simulation. This is a stochastic method.

Phases of QAM:
1. Problem identification: The problem must be precisely and concisely defined. In this step
not only the problem is defined but also uses, objectives and limitations of the study are
stressed in the light of the problem.
2. Model building: In this step the parameters or uncontrollable inputs of the problem must
be recognized, the decision variables or controllable inputs specified, and the objective
and constraints expressed quantitatively. If any of the uncontrollable inputs of the model
are subject to variation, the model is called a stochastic model otherwise the model is
referred to as a deterministic model. Mathematical models are built by translating verbal
statements into mathematical expressions involving the decision variables of the problem.
3. Solution of the model: In this step the mathematical model developed at earlier stage is
solved through scientific methods. Solution must satisfy the objective function as well as
constraints. There are various scientific methods or techniques available such as Linear
Programming, Transportation Model, Assignment Model, Game Theory, Queuing Theory,
and Statistical Methods etc.

6
4. Validation of the model: A model is said to be valid if it gives a reliable result (output) for
a set of inputs under the given conditions. Such validation is possible for a limited period
of time. As the time changes, original assumptions and conditions in which the model was
developed also change. Hence it is essential to check the validity of model from time to
time.
5. Report generation: After the model appears to give satisfactory results, a report must be
generated to convey the model’s solutions. This report should contain a statement of the
problem, the assumptions made and an indication of the general approach to problem
solution. Of course, a concise summary of a recommendation based on the model’s results
should be stated. The report should be written in such a way that the decision maker easily
understands it.

Models in QAM:
Models play a significant role in business decision making. A model is an abstract of
reality, a simplified version of something. It shows the relationship between cause and effect and
between objectives and constraints. An important step in the systems approach to decisions
making is the development of models which are used to obtain valuable insights into the behavior
of a system. Problems that defy direct solution because of size, complexity, or structure are often
assessable through model simulation. The purpose of model building is to attempt to describe the
essence of a situation or activity by abstracting from reality so that the decision maker can study
the relationship between relevant variables in isolation. A model is not intended for duplicating
the reality in all aspects, for models that duplicate reality reveal nothing. The key to model
building and problem solving lies in abstracting only the relevant variables that affect the criteria
and expressing the relationships in a testable form.
Model building is the essence of the QAM approach. But the classification of QAM models is
a subjective problem. However, models may be classified according to their functions, purposes,
degree of abstraction, procedure of solution etc.
The classifications of the models are as follows:

Criterion of Classification Categories of Models


1 Degree of Abstraction (a) Physical (b) Analogue (c) Mathematical
2 Purpose (a) Descriptive (b) Explanatory
(c) Predictive (d) Perspectives
3 Special Behaviour Characteristics (a) Static (b) Dynamic
(c) Linear (d) Non-linear
4 Degree of Certainty (a) Deterministic (b) Probabilistic
5 Procedure or Method of Solution (a) Analytical (b) Simulation
6 Form of Structure (a) Allocation (b) Inventory (c) Queuing
(d) Replacement (e) Competitive
1. Physical or Iconic model: This is the representation of the real object or situation. For an
example, child’s toy truck is a model of a real truck. Thus, structure of an atom, model of an
aeroplane, photograph of a machine, layout drawing of a factory etc. are examples of
physical model. These are also called static model.
2. Analogue model: These are abstract models mostly showing inter and intra relationships
between two or more parameters. Some examples are histogram, frequency table, flow chart,
Gantt chart, price-demand graph etc. These are also called dynamic model.
3. Mathematical model: Here sets of relations are represented in the form of mathematical
equations, using symbols to present various parameters.
4. Descriptive Model: The model which describes the situation under study is called descriptive
model. For example, a table showing various demand levels and their respective frequencies
is a descriptive Model.
7
5. Explanatory Model: A model which explains behaviour of various components of the system
under study is called explanatory model. For instance, correlation between share price and
earning per share may be termed as explanatory model.
6. Predictive Model: Predictive Model predicts future behaviour of various variables on the
basis of their present relationships; i.e. a model making a demand forecast at a given level
of price for a commodity.
7. Prescriptive Model: These models suggest the preferred course of action in a given situation
such as linear programming model or inventory model.
8. Static Model: Static models are concerned with determining an answer for a particular set of
fixed conditions that will probably not change significantly in the short run.
9. Dynamic Model: Dynamic models take account of time factor and admit the impact of
changes generated by time.
10. Linear Model: A linear model is one where each of its components is related to other
variables linearly. Most of the OR techniques presently available use linear models.
11. Non-Linear Model: A non-linear model is one in which one or more components of the
model exhibit non-linear behaviour.
12. Deterministic Model: In deterministic models, variables and their relationships are stated
exactly. Conditions of certainty and perfect knowledge are presumed to exist. Same results
are always obtained from the same data. The break-even model, widely used in business, is
one of the best examples of a deterministic model.
13. Probabilistic Model: Probabilistic models are used to deal with those situations in which
outcomes of managerial actions cannot be predicted with certainty. This implies that a
decision or strategy may result in one of several different payoffs, each with a certain
probability.
14. Allocation Model: The problem of allocation can arise whenever one must select the level
of certain activities which must compete for certain scarce resources necessary tp perform
those activities. This allocation is made on the basis of the goal to be achieved, such as
maximization of profits or minimization of costs. The linear programming model, the
transportation model, or the assignment models are examples of allocation models.
15. Inventory Model: Inventory models deal with a class of problems involving the storage of
idle resources until they are needed. That is, inventories involve what is apparently the
simplest operations that can be conceived – holding or storing resources. The decisions
required generally entail the determination of how much of a resource to acquire and how
to acquire it. A large number of resources may be involved – for example, in determining
how many of each of a large number of parts to purchase or produce and when to do so in
order to minimize total cost of managing, holding and storage.
16. Queuing Model: The formation of queues or waiting lines is a common phenomenon which
occurs whenever the current demand for a service – exceeds the current capacity to provide
that service. The objective of this model is to allow one to determine the optimum number
of personnel or facilities necessary to service customers who arrive at some random rate
when considering the cost of service and the cost of waiting. These types of problems are
also called waiting line problems.
17. Replacement Model: Replacement models serve to solve a class of problems in which
equipment or other assets must be replaced because of deterioration or complete failure.
These models are considered in two categories. The first deals with equipment that
deteriorates gradually, with time. The second category deals with items that have a more or
less constant efficiency with time. It often happens that items are replaced not because they
no longer perform to their designated standards, but because more modern equipment
performs to higher standards.
18. Competitive Model: Competitive situations are characterized by the fact that two or more
individuals are making decisions in situations that involve conflicting interests and in which
the outcome is controlled by the decisions of all the parties involved. Many conflicting
8
situations of this type are found in economic, social, political, and military problems. Game
theory, a conceptual framework of great importance, provides solutions for some of these
problems.
It should be kept in mind that the above classification is not exclusive in a model may belong to
more than one category. A linear programming model, for instance, is a static, deterministic,
linear, allocation model, all at the same time. An inventory model may be deterministic if we can
accurately predict the demand and cost figures but it is also probabilistic if demand control be
predicted with certainty or cost figures differ from time to time.

Advantages of Models:
A model can be conceptually regarded as a substitute for the real system. Thus, instead of
investigating and experimenting with the real system, we can interrogate the model usually with
less risk, less time and less money. A model can be helpful in any one of the following five ways:
(a) as an aid to thought (b) as an aid to communication (c) as a tool for prediction (d) for control
purposes and (e) for training and instruction. Rational decision making requires forecasted
performance. Models provide this link. As a system of prediction, models – represent scientific
method. The following advantages may be obtained from the use of models:
1. Through a model, the problem under consideration becomes controllable.
2. It provides some logical and systematic approach to the problem.
3. It indicates the limitations and scope of an activity.
4. Models help incorporating useful tools that eliminate duplication of methods applied to
solve any specific problem.
5. Models help in finding avenues for new research and development in a system.
6. Models are often most economical and safest way to test alternative actions, since models
provide a means of predicting future.
7. Models, if properly constructed, can suggest where a decision maker’s information is in
sufficient though the decision maker himself may not be aware of it.
8. Models can be manipulated easily.
9. Models take a complicated situation and make it simpler by eliminating all those factors
which in the decision maker’s judgment, are irrelevant to the problem to be considered.
10. Models provide a framework for decision maker. Thus, a number of diverse
considerations can be brought together in an organized fashion.
11. It determines the extent of risk and uncertainties involved among alternatives.
12. Models help to formulate policy and programs by keeping balance with changing
conditions.
13. Models provide different information by evaluating alternative course of action.
14. It facilitates in short-term profit planning.

Disadvantages of Models:
In spite of the advantages of models, there are also some limitations:
1. As models are abstractions and simplifications, they may inaccurately reflect the real
situation and one may fail to take into account all variables and expectations.
2. It is often difficult to define all the important elements of a model in mathematical terms
and set them down on paper.
3. At the end of the process of abstraction, the model can be so complex that it becomes very
difficult to distinguish between different elements accurately.
4. Models may sometimes be very expensive in relation to the expected returns from their
use.
5. There is on many occasions difficultly of communication with management and personnel
who do not understand the models and the result obtained.

9
6. There is no guarantee that an investment of time and effort in constructing the model will
play dividends in the form of satisfactory prediction.
7. It may be beyond the ability of a mathematician to manipulate the symbolic language so
as to obtain useful results.
8. It is very difficult to build representative models. Some scientists become so devoted to
their model (especially if it is a brain child) that they will insist that this model is the real
world.
9. Models do not consider the qualitative aspects and it gives no solution to the problem.
10. It requires adequate knowledge and experience in model building as well as in using
models and this versatility is not always available.

Principles of Applying Models:


The following principles are to be considered in using models:
1. To determine the pattern of decision in which model will be used.
2. The objectives of using models must clearly be expressed.
3. To decide about the person to whom the task of model building will be entrusted.
4. To appraise the success of model in attaining the objectives.
5. Models used in an organization should be acceptable to all.
6. The model builder should have sufficient knowledge about the strengths and limitations
of model.
7. The task of building model and the function of collecting data should be started at the same
time.
8. In order to construct or validate a model, the model maker must typically have access to
information.
9. Development in model is needed to keep balance with the changing situation.

Features of a Good Model:


The main properties of a good model may be listed as below:
1. A good model should be capable of taking into account new formulation without having
any significant change in its frame.
2. Assumptions made in the model should be as small as possible.
3. It should be simple and coherent. Number of variables used should be less.
4. It should be open to parametric type treatments.
5. A model must have one or more objectives, which must be stated explicitly.
6. The model will have variables, or unknowns, that represent the factors that can change
in the problem.
7. There should be interplay among variables and parameters as relationships, which are
explained in mathematical terms.
8. Model will have boundaries within which it must operate in order to retain its validity.
These are typically expressed as constraints on optimization of objectives.

Modeling Process:
There are five basic elements in the modeling process, namely, (a) Abstraction,
(b) Validation, (c) Prediction, (d) Evaluation and (e) Revision.

Limitations of QAM:
1. Quantification
2. High costs are involved in the use of QAM
3. Huge time involvement in using QAM.
10
4. QAM are just the tools of analysis and not the complete decision making process.
Applications for Quantitative Techniques in Business Decision Making
Quantitative techniques use surveys, tests, experiments and other data-gathering methods to
assemble information. The facts gathered in this way can be used to help make business decisions
about marketing, advertising, packaging, assembly techniques, financial decisions, which services
to offer and many other business considerations. Both gathering and analyzing information is
crucial to applying quantitative data to good business decisions.
Finances
One area where quantitative techniques are applied in business is in the area of finances.
Some of the models that financial managers and analysts use are return on investment,
decision trees and net present value. Financial analysts determine how much profit a
particular product brings in versus the costs of producing that product. They run regressions
and analyses to note trends over time and determine how much to invest in a particular
business line. Financial analysts also use quantitative methods to determine productivity and
whether or not to hire, retain or lay off workers. They use quantitative data to manage risk
and create investment vehicles.
Advertising
Advertisers use quantitative data to determine how many viewers or readers will see a
particular advertisement in a particular medium. They use data from rating services to find
out how many people click on a certain website or watch a particular television show at any
time. Advertisers also use quantitative data to do pre- and post-testing of advertisements.
Advertisers use surveys to test ad recall in viewers, and attitudes about proposed
advertisements, among other things.
Marketing
Companies make heavy use of statistics to determine how to market their products, which
markets their products and services will do best in and which consumers will buy their
products. There are thousands of companies in the United States that gather and analyze data
about consumer interests, desires, likes, dislikes, motivations and concerns. Marketers use
this data to focus sponsorships, direct mail campaigns and position their companies in the
general culture. Marketers also use data from UPCs (Universal Product Code) at stores (often
in combination with shopper discount cards) to determine who is buying their products, how
often and where. This also gives them important information to use in making decisions
about stocking, delivery and promotions.
Insurance
Insurance companies have a multitude of applications for quantitative data. Although many
of these applications could transfer to other businesses, insurance companies have dozens of
statisticians or actuaries on staff. Therefore, they have the manpower and know-how to
analyze mountains of data. For example, insurance companies gather data about each
salesperson in each line of business. They then analyze the data to see if there are similarities
in the top salespeople so they can recommend improvements to those not doing so well.
They also see which lines of business produce profit and which should be closed down
because they are unprofitable. Insurance company actuaries also analyze data on accidents,
fires, floods and other mishaps that require them to pay out money and use these analyses to
set insurance rates for their customers.

Conclusion:
The description of the meaning and use of various quantitative techniques clearly explains the
role of such techniques in the field of business and industry. Because of their usefulness the
study of QAM has gained increasing importance especially in modern times. There is growing
11
awareness amongst the people that unless they make themselves familiar with QAM, they
would not be able to understand and appreciate the problems of modern business units. With
computer facilities becoming widespread, the significance of QAM is likely to grow in the
upcoming years in spite of the above stated limitations. Not only that many new quantitative
techniques may develop to tackle the ever increasing problems of the business and industrial
world. However, these techniques should be taken only as an effective aid to decision making
process; they can never be a substitute for human skill, experience and judgment.

12
Function

Introduction: Function is a logical relationship. Following cause-effect rule phenomena or


variables are related by some rules. Functions explain the nature of correspondence between
variables indicated by some formula, graph or a mathematical equation. A function is a term used
to symbolize relationship between or among the variables. When two variables are so related that
for any arbitrarily assigned value to one of them there corresponds a definite value or a set of
definite values for the other, the second variable is said to be the function of the first. We shall
now introduce the some different types of functions, which are particularly useful in business and
economics.

Definition of Mathematical Functions:

Function: A function is a special relationship that may exist between variables. When two
variables are so related that for any arbitrarily assigned value to one of them there corresponds a
definite value (or a set of definite values) for the other, the second variable is said to be the function
of the first.
Very simply a function f is a rule that associates to any given number x a single number
f(x), read ‘f of x’. Here f(x) is the number that results when x is given, f(x) does not mean f times
x. For example, the function f which associates the square of a number to the given number x is
f ( x) = x 2
The given number x is called the independent variable and the number associated to it by
the function is called the dependent variable since it depends for its value on x.
Thus, a function is a mathematical relationship in which the values of a single dependent
variable are determined from the values of one or more independent variables.

Mapping: The equivalent expression for a function is mapping.

Variable: A variable is a quantity (or symbol) which can take various values. Let x be a symbol
used during any mathematical investigation to which may be assigned any numerical value out of
a given set of real numbers. Then x is called a variable or a real variable and the totality of the
values of x constitutes what is called the domain of x.
Variables frequently used in business and economics include price, profit, revenue, cost,
national income, consumption, investment, imports, exports and so on. For example we may
represent price by P, profit by  , revenue by R cost by C and so forth.

Independent Variable: A variable which may take any arbitrary value assigned to it is called an
independent variable.

Dependent Variable: A variable which assumes its value as a result of a second variable or a
system of variables taking up any set of arbitrary values assigned to them is called a dependent
variable.

Continuous variable: If x assumes successively every numerical value of an aggregate of all real
numbers from a given number ‘a’ to another given number ‘b’ then x is called a continuous real
variable.

Domain: The set of values that are to be used for the independent variable is called the domain of
the function or simply domain.
13
Suppose the variable x assumes all the numbers between two given numbers a and b. All
numerical values of x between a and b taken collectively from the field of variation of x between
a and b and it is called the domain of x.
Further if a and b are included in the domain (interval) then the domain is called a closed
domain or closed interval. It is denoted by a  x  b .
If both a and b are excluded in the domain (interval) then the domain is called a open
domain or open interval. It is denoted by a  x  b .
If one of the end points, say a is not included in the domain (interval) (a, b) then we say
that interval is partly closed and it is denoted by a  x  b which is a domain open at the left side.

Range: The set of all values of the dependent variable that correspond to the values of the
independent variable is called the range of the function or simply range.

Constants: A constant is a quantity which remains unchanged during any mathematical


operations or any investigation. There are two types of constants, namely, absolute constant and
arbitrary constant.

Absolute constant: Quantities which have the same value in all operations (or under all
circumstances) are called absolute constant e .g. 1, 5, 9, e, π etc.

Arbitrary constant: Quantities which have the same value under one investigation but are
different for different investigation are called arbitrary constant. It is a constant that is variable.
To identify its special status, we give it the distinctive name parametric constant or simply
parameter. e. g. in the equation of the straight line x cos  + y sin  = p , α and p are same for the
same straight line but they are different for different straight lines.

Absolute Value: The absolute value (or modulus) of a quantity x given always magnitude or
numerical value with positive sign. It is denoted by x . The absolute value x of a quantity x is
defined as follows:
 x if x  0

x =  0 if x = 0
 - x if x  0

Thus, if x  0 , x is positive since the negative of a negative number is a positive (absolute) value.
Properties of absolute values:
(i) If x and y are any two real quantities then x + y  x + y
(ii) If x and y are any two real quantities then x − y  x − y

Classification of Functions:
Any given function is either algebraic or transcendental.

Algebraic Functions:
14
A function is said to be an algebraic function which consists of a definite number of terms
involving only the operations of addition, subtraction, multiplication, division, root extraction and
raising to powers. Algebraic functions are two types, namely, polynomial function and rational
functions.

Polynomial function: A function of the type y = a 0 + a1 x + a 2 x 2 + .............. + a n x n , where


a o , a 1 , a 2 , ............., a n are all constants and n is a positive integer, then y is called a
polynomials in x of degree n.

Rational functions: A function which appears as a quotient of two polynomials such as


a 0 + a1 x + a 2 x 2 + .............. + a n x n
is called a rational function. If P(x) and Q(x) are two
b0 + b1 x + b2 x 2 + .............. + bn x m
P( x)
polynomials the ratio y = where P(x) and Q(x) have no common factor, said to be a
Q( x)
rational function.

Irrational functions: Irrational function is an algebraical function of x when it involves root


extraction of terms involving x.
e.g. x+2, ( x 2 + 2 + 3 etc are the examples of irrational functions.

Transcendental Functions:
Functions which are not algebraic are said to be transcendental. The transcendental functions are:
Trigonometric functions, Inverse trigonometric functions, Exponential functions and Logarithmic
functions.
Trigonometric Functions: y = sin x, y = cos x, y = tan x etc are all Trigonometric
functions of x.
Inverse trigonometric Functions: y = sin −1 x, y = cos −1 x, y = tan −1 x etc are all inverse
trigonometrical functions of x or inverse circular functions of x.
Exponential Functions: y = 2 x , y = 10 x , y = x x , y = e x etc. are all exponential functions
of x.
Logarithmic Functions: y = log e x, y = log10 x, y = log a x etc. are all logarithmic functions
of x.
Explicit Functions: A function which is directly expressed in terms of the independent
variable is called an explicit function e.g. y = sin x, y = r cos  .
Implicit Functions: A function which is not expressed directly in terms of the independent
variable is called an implicit function e.g. x 2 + y 2 = a 2 , ax 2 + 2hxy + by 2 = 0 .
Periodic Functions: Exponential and Trigonometrical functions are called periodic
functions. If f ( x) = f ( x + d ) for all values of x for which the function is defined. f (x) is
called a periodic function with the period d. Trigonometrical functions are periodic with 2
2 i
as a period, Exponential function is periodic with e as period.
Single valued Functions: A function y is said to be a single valued function when it has
only one value corresponding to each value of the independent variable, e.g.
y = x 2 , y = sin x etc .
Many valued Functions: A function y is said to be a many valued function when it has
more values than one corresponding to each value of the independent variable, e.g.
y 2 = x, y = sin −1 x etc .
15
Odd function: A function is said to be odd function if it changes sign with the change of sign
of the variable. Let f (x) be a function. If we replace x by − x then f (− x) = − f ( x)
Even Function: A function is said to be an even function if the function does not changes sign
with the change of the variable. Let f (x) be a function. If we replace x by − x then
f (− x) = f ( x)
Monotone function: A function f (x) is said to be monotonic in a given interval (a, b) such
that for two values x1 and x 2 x 2  x1 of the variable x in the interval either (i) f ( x 2 )  f ( x1 )
or (ii) f ( x 2 )  f ( x1 ) .
If f ( x 2 )  f ( x1 ) , then the function f (x) is continually increasing. The function f (x) is
called a monotonically increasing function.
Again, if f ( x 2 )  f ( x1 ) , then the function f (x) is continually decreasing. The function f (x)
is called a monotonically decreasing function.
Definition of Business and Economics Functions:
Demand Function: Demand functions are an essential concept in the study of economics. Usually
these functions are curves rather than straight lines, but straight lines provide good illustrations of
demand characteristics. The demand function specifies the amounts of a particular commodity
that buyers are willing and able to purchase at each price in a series of possible prices during a
specified period of time. Demand refers to the relationship between price and quantity.
Conventionally, we plot price on the vertical axis and quantity demanded on the horizontal axis.
Let p be the price and x be the quantity demanded, the function x = f (p) is plotted as a demand
curve. It usually slopes downwards as demand decreases when prices are increased. p

O x
Supply Function: Supply function specifies the amounts of a particular product that a producer
is willing and able to produce and make available for sale at each price in a series of possible
prices during a specified period of time. Let p be the price and x be the quantity supplied; the
function x = g ( p ) is plotted as a supply curve. When price increases quantity supplied
increases, therefore, a supply curve slopes upward
P

O x

Total Revenue (TR): A firm’s total revenue that is derived from the sale of a product is given by
price multiplied by quantity. It can be expressed mathematically, TR = p  q.
From the demand function we observe that price is a function of quantity,
i.e. p = f (q ) .
Total revenue is thus represented as a function of quantity.
i.e. TR = f (q )  q .

16
Total Cost (TC): The cost of production to the firm depends upon the costs of inputs used in the
production process and the quantity of product manufactured. If x is the quantity produced of a
certain good by a firm at total cost C, we can write the total cost function TC = f (x) .
It may be noted that the cost TC of producing goods can be analyzed into two parts: (i)
fixed cost which is independent of x with certain limits and (ii) variable cost depending on x. Thus,
we may have cost function of the type, TC = FC + VC , where FC is fixed cost and VC is variable
cost.
Cost curves are obtained from the knowledge of production functions. Usually, the cost
curve is rising to the right as the cost of production generally increases with the output (x).

Total Profit: Profits of a firm depend upon both revenue and cost. Profits are defined as the excess
of total revenue over total costs. Symbolically it can be expressed as, P = TR − TC .

Average Revenue (AR): The average revenue from a product is found by dividing the total
revenue by the quantity of the product sold. The function that describes average revenue is the
quotient of the total revenue function and the quantity.
We see that average revenue function and the demand function are equivalent.
TR f (q ).q
AR = = = f (q)
q q

Average Cost (AC): Average cost of production or cost per unit is obtained by dividing total cost
by the quantity produced.
C
AC =
x
Marginal Revenue (MR): Marginal revenue is the additional revenue derived from selling one
more unit of a product or service. If each unit of a product sells at the same price, the marginal
revenue is always equal to price.
d (TR )
Thus, MR = ; the rate of change of revenue with respect to units of output.
dq

Marginal Cost (MC): Marginal cost is defined as the change in total cost incurred in the
production of an additional unit.
d (TC )
MC = ; the rate of change of cost with respect to units of production.
dq

Marginal Profit (MP): Marginal profit analysis is concerned with the effect on profit if one
additional unit of a product is produced and sold. As long as the additional revenue brought in by
the next unit exceeds the cost of producing and selling that unit, there is a net profit from producing
and selling that unit and total profit increases. If, however, the additional revenue from the selling
the next unit is exceeded by the cost of producing and selling the additional unit, there is a net loss
from that next unit and total profit decreases.
d (TP )
MP = ; the rate of change of profit with respect to units of output.
dq
A rule of thumb concerning whether or not to produce an additional unit (assuming profit
maximization is of greatest importance) is given below.
(i) If MR  MC , produce the next unit.
(ii) If MR  MC , do not produce the next unit.
(iii) If MR = MC , the total profit will be maximized.

17
Production Function: A production function is a technical relationship between the inputs of
production and the output of the firm’s. The relationship is such that the level of output depends
upon the level of inputs used, not vice versa. A production function can be written as Q = f ( L, K )
, where L and K are quantities of labour and capital respectively required to produce Q.
In Economics, the Cobb – Douglas production function defined as
Q = aL K  , where  +  = 1 .
Utility function: The term utility refers to the benefit or satisfaction or pleasure of a person gets
from the consumption of a commodity or service. In abstract sense, utility is the power of a
commodity to satisfy human want i.e. utility is want – satisfying power. A commodity is likely to
have utility if it can satisfy a want. For example, bread has the power to satisfy hunger, water
quenches our thirst and so on.
If U ( x, y ) denotes the satisfaction obtained by an individual when he buys quantities x and
y of two commodities X and Y, then U ( x, y ) is called the utility function or utility index of the
individual.

Consumption function: If C is the total consumption of the community dependent on income Y


and propensity to consume c the aggregate consumption function is defined by
C = a + cY
But since Y = C + S
S = Y − (a + cY ) ; This is the savings function of the community.

Relation between Average Cost (AC) and Marginal Cost (MC):


Let us assume that total cost, C = f (x)
C
Thus, AC = .
x
dC
x −C
d d C 1 dC C 1
( AC ) = ( ) = dx 2 = ( − ) = ( MC − AC )
dx dx x x x dx x x

Case 1: When average cost curve slopes downwards i.e. when AC is declining, its slope will be
negative. AC
MC AC
d
( AC )  0 MC
dx
1
( MC − AC ) < 0 O x
x
MC − AC  0
MC < AC
Thus when AC curves slopes downwards MC curve will lie below AC curve.

Case 2: When AC curve reaches a minimum point, its slope becomes zero.

d
( AC ) = 0 AC MC AC
dx
MC

18
1
( MC − AC ) = 0
x
MC – AC = 0
MC = AC

O x
Thus MC curve and AC curve intersect at the point of minimum average cost.

Case 3: When average cost curve slopes upward


d
( AC )  0 AC MC
dx
MC AC
1
( MC − AC ) > 0
x
MC − AC  0
MC >AC
O x

Thus when AC curve slopes upward MC curve will be above AC curve.

Relationship between Average Revenue (AR) and Marginal Revenue (MR):


When average revenue curve slopes downwards i.e. when AR is declining, its slope will be
negative. Thus when AR curves slopes downwards MR curve will lie below AR curve.
When AR is always same then there is no difference between AR and MR. This relationship will
be applicable for pure competitive market.
AR, MR
AR, MR

AR=MR

MR AR
O x O x

Pure competitive market Imperfect competitive market


Example 1: If f ( x) = x + 2 x . Find f (1), f (0), f (−1), and f (a )
2

Solution: Given that f ( x) = x 2 + 2 x


f (1) = (1) 2 + 2.1 = 3
f (0) = (0) 2 + 2.0 = 0
f (−1) = (−1) 2 + 2(−1) = −1
f ( a ) = ( a ) 2 + 2a
1
Example 2: Determine the domain of the function y =
x −1
1
Solution: The denominator x − 1  0 because is not a real number.
0
x 1
The domain of the function is all values of x except 1.
19
x
Example 3: Determine the domain of the function y =
x
Solution: When x = 0 , then the function is undefined. All of the values of x are in domain of the
function except x = 0 i.e. (−, 0)  (0, )
x
Example 4: Determine the domain of the function f ( x) =
x+2
Solution: At x is negative then f (x) becomes imaginary. Hence the domain is all positive real
values of x including zero i.e. 0  x  

x2 − 9
Example 5: Find the domain and range of the function f ( x) =
x−3
Solution: When x = 3 , then the function is undefined. All of the values of x are in domain of the
function except x = 3 .
The domain of the function is (−, 3)  (3, )
The range of the function is (−, 6)  (6, )

Example 6: A Company has determined that the total cost of producing x prices of plates per
month depends on both its fixed cost and variable cost. If the fixed cost is per month is Tk.6000
and the variable cost per month is 3x + 0.5 x 2 express the total cost C(x) in terms of the fixed cost
and variable cost. Using the cost function determine the cost of producing 500 plates.
Solution: Total cost = Fixed cost + Variable cost
C ( x) = 6000 + 3x + 0.5 x 2
To produce 500 plates the cost will be
C (500) = 6000 + 3(500) + 0.5(500) 2
= 6000 + 1500 + 125000
= 132500

Example 7: y = 10 x + 500 where x represents the advertising cost and y represents the sales. Find
the domain and range.
Solution: Since x is advertising cost so it cannot be negative and hence the domain of the given
function is all the non-negative values of x and the range is all values between 500 and  .

Example 8: The anticipated price of a product is Tk.15. The fixed cost of manufacturing the
product is Tk.10,000 and the variable cost is Tk.10 per unit. Plant limitation is restricted
production to 5000 units. Develop revenue and total cost functions and calculate breakeven
production. Find also the profit function.
Solution: The revenue function is R = 15 x for 0  x  5000
and the total cost function is TC = 10,000 + 10 x for 0  x  5000 .
Breakeven point is found by equating the two functions i.e. revenue function and total cost
function.
R = TC
15 x = 10,000 + 10 x
5 x = 10,000
x = 2000
Further, profit is defined as revenue minus total cost.
Profit is thus P = R − TC
P = 15 x − (10,000 + 10 x)

20
P = 5 x − 10,000

Example 9: Let the unit demand function be x = ap + b and the cost function be c = ex + f where
x = sales (in units), p = price (in Tk.), f = fixed cost (in Tk.), e = varible cost, b = demand when p
= 0, a = slope of unit demand function.
Required: (i) find the cost C as a function of p.
(ii) find the revenue function R (x).
(iii) find the profit function P (x).

Solution: (i) Cost, c = ex + f


= e(ap + b) + f .
(ii) Revenue = price  quantity
=p  x
x b x 2 bx
= ( − ).x = −
a a a a
(iii) Profit = Revenue – Cost
x 2 bx
P (x) = ( − ) – (ex + f )
a a
2
x bx
= − − ex − f
a a
2
x b
= − ( + e) x − f .
a a
Example 10: Find the average cost and marginal cost if total cost TC = 1000 + 100q – 10q2 +
q3
TC 1000 + 100q − 10q 2 + q 3
Solution: Average cost (AC) = =
q q
1000
= + 100 − 10q + q 2
q
d
Marginal cost (MC) = (TC)
dq
d
= (1000 + 100q − 10q 2 + q 3 )
dq
= 100 - 20q + 3q 2
Example 11: If the demand function of the monopolist is 3q = 98 − 4 p and average cost is
3q + 2 where q is output and p is the price, find maximum profit of the monopolist.
Solution: Given average cost, AC = 3q + 2
Total Cost, TC = AC x q = ( 3q + 2 ). q = 3q 2 + 2q
d
Marginal cost (MC) = (TC )
dq
d
MC = (3q 2 + 2q ) = 6q + 2
dq
Again given that, 3q = 98 − 4 p
98 − 3q
p=
4
Total Revenue (TR) = price x quantity

21
TR = p x q.
98 − 3q 98q − 3q 2
TR = .q =
4 4
d 98q − 3q 2
Marginal Revenue (MR) = ( )
dq 4
98 6q
= −
4 4
We know that under monopoly market, profit will be maximum at MC = MR.
98 6q
6q + 2 = −
4 4
24q + 8 = 98 − 6q
30q = 90
q = 3.
So the maximum profit of the monopolist will be obtained at q = 3.
Again Total Profit (TP) = TR – TC
98q − 3q 2
TP = ( ) - ( 3q 2 + 2q )
4
When q = 3, then the profit will be maximum
(98)(3) − 3(3) 2
i.e. maximum profit = − 3(3) 2 − 2(3) = 33.75
4

Example 12: Show that marginal cost (MC) must equal marginal revenue (MR) at the profit
maximizing level of output.
Solution: We know that, Total profit = Total revenue – Total cost
i.e. TP = TR – TC
d (TP )
To maximize TP, must equal zero.
dQ
d (TP ) d (TR ) d (TC )
= − =0
dQ dQ dQ
d (TR ) d (TC )
= .
dQ dQ
MR = MC (showed)

Example 13: If the cost function is C ( x) = 4 x + 9 and the revenue function is R( x) = 9 x − x 2 ,


where x is the number of units produced (in thousands) and R and C are measured in million of
Tk., find the following:
(i) Marginal revenue.
(ii) Marginal revenue at x = 5 .
(iii) Marginal cost.
(iv) The fixed cost.
(v) The variable cost at x = 5 .
(vi) The break-even point, that is R( x) = C ( x) .
(vii) The profit function.
(viii) The most profitable output.
(ix) The maximum profit.
(x) The marginal revenue at the most profitable output.
(xi) The revenue at the most profitable output.

22
(xii) The variable cost at the most profitable output.

Solution: Given that, R( x) = 9 x − x 2


C ( x) = 4 x + 9
d d
(i) MR = ( R) = (9 x − x 2 ) = 9 − 2 x
dx dx
(ii) When x = 5 , then MR = 9 − 2  5 = −1 .
d d
(iii) MC = (C ) = (4 x + 9) = 4
dx dx
(iv) The fixed cost FC = 9.
(v) When x = 5 , the variable cost (VC) is 4  5 = 20.
(vi) For break – even point, R(x) = C(x).
9x − x 2 = 4x + 9
x 2 − 5x + 9 = 0
5  − 11
x= .
2
(vii) Profit, P = R – C = (9 x − x 2 ) - (4 x + 9) = 5 x − x 2 − 9 .
(viii) Here profit, P = 5 x − x 2 − 9
dP
= 5 − 2x
dx
dP
For maximum or minimum, =0
dx

5 − 2x = 0
5
x= .
2
d 2P 5
Again 2
= −2 ; which is negative. So the profit function is maximum at x = .
dx 2
5
Thus, the required most profitable output is x = .
2
(ix) The maximum profit, P = 5 x − x − 9 .
2

5 5 11
= 5  − ( ) 2 − 9 = − ; which shows a loss.
2 2 4
5 5
(x) When x = , then MR = 9 − 2 x = 9 − 2  = 4 .
2 2
5 5 5
(xi) When x = , then R = 9 x − x 2 = 9  − ( ) 2 = 16.25 .
2 2 2
5 5
(xii) When x = , then variable cost VC = 4 x = 4  = 10
2 2

Equations

23
Introduction: The practical use of the equations and inequalities are in evolving certain
relations and finding out the value of the unknown. Sometimes complicated verbal statements
when translated into equations or inequalities can be solved with great ease.

Equation:
A mathematical statement setting two algebraic expressions equal to each other is called an
equation. Simply, an equation states the equality of two algebraic expressions. The algebraic
expressions may be stated in terms of one or more variables. For example,
2 x − 1 = 0, x + y = 6 and x + y + z = 100 are equations with one, two and three variables
respectively. The solution of an equation consists of those numbers which, when substituted for
the variables, make the equation true. The numbers, or values of the variables, which make the
equation true are referred to as the roots of the equation. We say that the roots are the values of
the variables, which satisfy the equation.

The degree of an equation is denoted by the highest index of the variable in any equation.
An equation with the highest index or power as one is of the first degree. It is also called a linear
equation since its graph represents a straight line.

The higher degree equations are also called higher degree polynomials or polynomial
equations. An equation having its highest index as two is called the quadratic equation. For
example x 2 + 3 x + 6 = 0 is quadratic equation in one variable. But the equations
x 2 + y 2 = 16 and x 2 + xy + y 2 = 20 are quadratic equations in two variables. Further, higher
order equations are cubic with highest index of the variable 3. For example x 3 + 6 x 2 + 12 x + 7 = 0
is a cubic equation in one variable. Similarly, higher order equations are bi-quadratic with the
highest index of the variable 4. For example x 4 + 8 x 2 + 7 x = 0 is a bi-quadratic equation in one
variable.

Linear Equation: An equation in which all variables are raised to the first power is known as a
linear equation. Linear equation is the simplest form of equation.

Characteristics of linear equation:


(i) The highest exponent/index of the variable (s) of a linear equation is one.
(ii) The general form of a linear equation is ax + by + c = 0
(iii) The graph of a linear equation always shows a straight line i.e. a linear equation must
not have any bend (or turning point).
(iv) Linear equation has one root or solution for each variable (s).

Differences between linear and Non-linear equations:

Linear equation Non-linear equation


The highest exponent or power of a The lowest power of a variable of a non-
variable of linear equation is one. linear equation is at least two.

24
The general form of a linear equation The general form of a non-linear
is ax + by + c = 0 equation is
n −1 n−2
a 0 x + a1 x + a 2 x + ....... + a n x = 0
0 n

The graph of a linear equation is The graph of a non-linear equation is


straight line. curve.
Linear equation has no bend. A non-linear equation has at least one
bend.
Variable (s) has single root or Variable (s) has at least two roots.
solution.

Identities: When equalities hold true whatever be the value of the variables, they are called
identities. For example, ( a + b) 2 = a 2 + 2ab + b 2 .

Inequality: An inequality is an expression relating the condition that either two quantities are not
equal or that they may or may not be equal. The first is a strict inequality and involves the
conditions greater than or less than; the latter involves the conditions ‘equal to or greater than’ or
equal to or less than’.
Any quantity a is said to be greater than another quantity b when a – b is positive. Thus, 5
is greater than – 3, because 5 - (- 3) or 8 is positive. Also b is said to be less than a when b – a is
negative. Thus – 5 is less than – 3 because – 5 - (- 3) or – 2 is negative. In accordance with the
definition of inequality, zero must be regarded as greater than any negative quantity.
Examples of inequalities are 3x + y  15, x  y + 5, x + 3 > 12, x + y < x – 5 etc.

Properties of Inequalities: The fundamental rules of inequalities are as follows:


(i) The same number may be added to or subtracted from both sides of the inequality and
the original sense of the inequality will be preserved.
Let a, b and c are real numbers and if a > b, then a  c > b  c.
(ii) The same positive number may multiply or divide both sides of the inequality and the
original sense of the inequality will be preserved.
Let a, b and c are real numbers and if a > b, and c is positive; then ac > bc or
a b
 .
c c
(iii) The same negative number may multiply or divide both sides of the inequality and the
sense of the inequality will be reversed.
Let a, b and c are real numbers and if a > b, and c is negative; then ac < bc or
a b
 .
c c
(iv) Any side of an inequality can be replaced by an equivalent expression.
(v) If both sides of the inequality are positive or negative, taking the reciprocal of each
side will reverse the sense of the inequality.
1 1
Let a and b are real numbers and if a > b, then  .
a b
(vi) If both sides of the inequality are positive and are raised to the same positive power,
the sense of the inequality remains the same.
Let a and b are real numbers and if a > b, and n is positive number, then an > bn.
Exercise: If a, b, c denote positive quantities, prove that
a 2 + b 2 + c 2  bc + ca + ab;
and 2(a 3 + b 3 + c 3  bc(b + c) + ca(c + a) + ab(a + b)

25
Solution: We know that, square of every real quantity is positive and therefore greater than zero.
Thus (a – b)2 is positive.
a2 – 2ab + b2 > 0;
a2 + b2 > 2ab……………(i)
Similarly, b2 + c2 > 2bc…………….(ii)
c2 + a2 > 2ca………….. (iii)
Adding (i), (ii) and (iii) we get,
2(a 2 + b 2 + c 2 )  2( bc + ca + ab)
 a 2 + b 2 + c 2  bc + ca + ab
Again, from (i) we can write, a2 – ab + b2 > ab
(a + b)(a2 – ab + b2)> ab(a + b)
a3 + b3 > ab(a + b)…………………(iv)
b3 + c3 > bc(b + c)…………………(v)
c3 + a3 > ca(c + a)…………………(vi)
By adding (iv), (v) and (vi) we get,
2(a 3 + b 3 + c 3  bc(b + c) + ca(c + a) + ab(a + b)

Quadratic Equation:
An equation which when reduced to the rational integral form contains the square of the unknown
quantity and no higher power is called a quadratic equation or an equation of the second degree.
For example ax2 + bx + c = 0 is a quadratic equation in one variable.
The graphic presentation of a quadratic equation takes the form of a parabolic, which are
a smooth and more or less a cup-shaped curve. This may often upwards or downwards depending
on whether ‘a’ which is the co-efficient of x2 in the above equation has a plus or a minus sign.

Solution Methods:
(i) Method of factorization.
(ii) Method of completing a perfect square.

Exercise: Solve the following equation:


5 x 2 − 11x − 12 = 0
Solution: 5 x 2 − 11x − 12 = 0
5 x 2 − 15 x + 4 x − 12 = 0
5 x( x − 3) + 4( x − 3) = 0
( x − 3)(5 x + 4) = 0
Either x − 3 = 0 or 5 x + 4 = 0
x = 3, − 4
5

Exercise: Solve the following equation:


ax 2 + bx + c = 0
Solution: ax 2 + bx + c = 0
b c
x2 + x + = 0
a a
b b2 c b2
x + 2.x. + 2 + − 2 = 0
2

2a 4a a 4a
2
b b c
(x + )2 = 2 −
2a 4a a

26
b 2 b 2 − 4ac
(x + ) =
2a 4a 2
b b 2 − 4ac
(x + ) = 
2a 4a 2
b b 2 − 4ac
x=− 
2a 4a 2
−b b 2 − 4ac
x= 
2a 2a
− b  b 2 − 4ac
x=
2a

Simultaneous Equation System:


A system of simultaneous equations is helpful for finding unique values for the unknowns. The
number of equations should be equal to the number of unknowns. However, the equations can be
of varying degrees. The following methods are generally used to solve the simultaneous
equations, when both equations are linear:
(i) Method of Substitution
(ii) Method of Elimination
(iii) Method of Cross Multiplication.

Method of Substitution:
The Substitution Method involves solving one of the equations for one of its variables (we usually
choose the easiest variable to solve for) and then substituting that expression in the other equation.
The resulting equation then contains only one variable and is easily solved.

Exercise: Solve the following system of equations by using method of substitution:


x − 2y = 6
x + 2 y = 10
Solution: Given that,
x − 2 y = 6............................(i )
x + 2 y = 10...........................(ii )
From equation (i) we get, x = 2 y + 6................(iii )
Substituting x = 2 y + 6 in the equation (ii) and we get
2 y + 6 + 2 y = 10
4 y + 6 = 10
4y = 4
y =1
Now substituting y = 1 in equation (iii) we get x = 8
Hence the solution is x = 8 , y = 1 .

Exercise: Solve the following system of equations by using method of substitution:


x − 2y = 8
3x + y = 3
Solution: Given that,

27
x − 2 y = 8.......................(i )
3 x + y = 3........................(ii )
From equation (ii) we get, y = 3 − 3 x................(iii )
Substituting y = 3 − 3 x in the equation (i) and we get
x − 2(3 − 3x ) = 8
x − 6 + 6x = 8
7 x = 14 x=2
Now substituting x = 2 in equation (iii) we get y = −3
Hence the solution is x = 2 , y = −3 .

Exercise: Solve the following system of equations by using method of substitution:


x − 2y = 1
2 x + y = −3
Solution: Given that,
x − 2 y = 1..............(i )
2 x + y = −3............(ii )
From equation (i) we get, x = 2 y + 1................(iii )
Substituting the value of x in (ii) we get, 2(2 y + 1) + y = −3
4 y + 2 + y = −3
5 y = −5
y = −1
Putting the value of y = −1 in (iii) and we get, x = −1 .
Hence the solution is x = −1 , y = −1 .

Exercise: Solve the following system of equations by using method of substitution:


2x + 6 y = 5
4x − 3y = 0
Solution: Given that,
2 x + 6 y = 5..............(i )
4 x − 3 y = 0............(ii )
5
From equation (i) we get, x = − 3 y................(iii )
2
5
Substituting x = − 3 y in equation (ii) we get
2
5 
4 − 3 y  − 3 y = 0
2 
10 − 12 y − 3y = 0
10 − 15 y = 0
y = 2/3

2
Substituting y = in equation (iii) we get
3
5 2
x = − 3 
2 3

28
5
x= −2
2
1
x=
2
1 2
Hence the solution is x = , y=
2 3

Method of Elimination:
There is another algebraic method for solving a system of equations. It is called the Elimination
Method and is based on the property that equals added to equals produce equals. Using this
property we can add the corresponding sides of two given equations in two variables to produce
one equation in one variable.

Exercise: Solve the following system of equations by using method of elimination:


4x + 5 y = 3
3x − 2 y = 8
Solution: Given that,
4 x + 5 y = 3...................(i )
3 x − 2 y = 8.....................(ii )
In this case, both equations must be multiplied by a suitable number in order to force either the
variable x or the variable y to drop out. We decide to multiply the equation (i) by 2 and the equation
(ii) by 5, thereby eliminating the variable y when the equations are added.
8 x + 10 y = 6............................(iii )
15 x − 10 y = 40..........................(iv)
Adding (iii) and (iv) we get, 23 x = 46
x = 2
Now, putting the value of x = 2 in equation (i) we get, y = −1 .
Hence the solution is x = 2 , y = −1 .
Exercise: Solve the following system of equations by using method of elimination:
5x − y = 3
3x + 2 y = 7
Solution: Given that,
5 x − y = 3.............(i )
3 x + 2 y = 7............(ii )
Multiplying equation (i) by 3 and (ii) by 5, we get,
15 x − 3 y = 9.............(iii )
15 x + 10 y = 35............(iv)
Subtracting (iv) from (iii) we get,
− 13 y = −26
y=2
Now, putting the value of y = 2 in equation (i) we get, x = 1 .
Hence the solution is x = 1 , y = 2 .
Exercise: Solve the following system of equations by using method of elimination:
2x + 3y = 2
4x + 9 y = 9
Solution: Given that,

29
2 x + 3 y = 2.............................(i )
4 x + 9 y = 9.............................(ii )
Multiplying equation (i) by 2 and (ii) by 1, we get,
4 x + 6 y = 4....................(iii )
4 x + 9 y = 9...................(iv)
Subtracting (iv) from (iii) we get,
− 3 y = −5
5
y=
3
5 −3
Now, putting the value of y = in equation (i) we get, x = .
3 2
−3 5
Hence the solution is x = , y= .
2 3
Method of Cross Multiplication:
Exercise: Solve the following system of equations by using method of Cross Multiplication:
5x − y = 3
3x + 2 y = 7
Solution: Given that,
5x − y − 3 = 0
3x + 2 y − 7 = 0
By the rule of cross of multiplication we get,
x −y 1
= =
7 + 6 − 35 + 9 10 + 3
x y 1
= =
13 26 13
Hence the solution is x = 1 , y = 2 .
Exercise: Solve the following system of equations:
3x + y = 0
2x + 3y − z = 8
4 x − y + 5 z = 16
Solution: Given that,
3 x + y = 0..................(i )
2 x + 3 y − z = 8................(ii )
4 x − y + 5 z = 16...............(iii )
Multiplying equation (ii) by 5 and (iii) by 1, we get,
10 x + 15 y − 5 z = 40..................(iv)
4 x − y + 5 z = 16.......................(v)
Adding (iv) and (v) we get,
14 x + 14 y = 56
x + y = 4............................(vi)
2 x = −4
Subtracting (vi) from (i) we get,
x = −2
Putting the value of x in (vi) we get, y = 6
Again putting the value x and y in (ii) and we get, z = 6

30
x = −2

Hence, the solution is y = 6 
z = 6 
Exercise: Solve the following system of equations:
x 2 + y 2 = 29
x− y =3
Solution: Given that,
x 2 + y 2 = 29..................(i )
x − y = 3.....................(ii )
From (ii) we get, x = y + 3.....................(iii )
Substituting x = y + 3 in equation (i) we get
( y + 3) 2 + y 2 = 29
y 2 + 6 y + 9 + y 2 = 29
2 y 2 + 6 y − 20 = 0
y 2 + 3 y − 10 = 0
( y + 5)( y − 2) = 0
Either ( y + 5) = 0 or, ( y − 2) = 0
y = −5 , or y = 2
when y = −5 , x = −2
when y = 2 , x = 5
x = −2  x = 5
Hence, The solution is  or 
y = −5 y = 2
Exercise: Solve the following system of equations:
x y
+ =5
2 5
2 5 5
+ =
x y 6
Solution: Given that,
x y
+ = 5......................(i )
2 5
2 5 5
+ = ....................(ii )
x y 6
From (i) we get, 5 x + 2 y = 50
50 − 2 y
x= ................(iii )
5
Putting the value of x in (ii) we get,
10 5 5
+ =
50 − 2 y y 6

10 y + 250 − 10 y 5
=
y (50 − 2 y ) 6
250 5
=
50 y − 2 y 2
6

31
250 y − 10 y 2 = 1500
y 2 − 25 y + 150 = 0
y 2 − 15 y − 10 y + 150 = 0
( y − 15)( y − 10) = 0
y = 15, 10
From (iii) we get,
x = 6 (when y = 10)
x = 4 (when y = 15)
x=6  x=4 
Hence, The solution is  or 
y = 10 y = 15
Exercise: Solve the following system of equations:
x y 3
− =
y x 2
x− y =3
Solution: Given that,
x y 3
− = ...................(i )
y x 2
x − y = 3....................(ii )
x− y 3
From (i) we get, =
xy 2

3 3
= [Using (ii)]
xy 2
xy = 2
xy = 4.................(iii )
From (ii) and (iii) we get, (3 + y ) y = 4
y 2 + 3y − 4 = 0
y2 + 4y − y − 4 = 0
y ( y + 4) − 1( y + 4) = 0
( y + 4)( y − 1) = 0
y = 1, − 4
Putting the value of y in (iii) we get,
x = 4 when y = 1
x = -1 when y = -4
Thus the solution is
x = 4 x = -1 
 or 
y = 1 y = −4

Exercise: Solve the following system of equations:


x( x + y + z ) = 10
y ( x + y + z ) = 40
z ( x + y + z ) = 50

32
Solution: Given that,
x( x + y + z ) = 10....................(i )
y ( x + y + z ) = 40..................(ii )
z ( x + y + z ) = 50...................(iii )
( x + y + z ) 2 = 100
By adding (i), (ii) and (iii) we get,
( x + y + z ) = 10.....................(iv)
Dividing (i), (ii) and (iii) by (iv) respectively and we get,
x = 1
y = 4
z = 5
Exercise: Solve the following system of equations:
3 x .9 y = 81
2x − y = 8
Solution: Given that,
3 x .9 y = 81...............(i )
2 x − y = 8..................(ii )
From (i) we get, 3 .3 = 3 4
x 2y

3 x+2 y = 34
x + 2 y = 4.........................(iii )
By solving (ii) and (iii) we get, x = 4, y = 0 .
Exercise: Solve the following system of equations:
8 y x − y 2 x = 16
2x = y2
Solution: Given that,
8 y x − y 2 x = 16................(i )
2 x = y 2 .........................(ii )
From (i) we get, y 2 x − 8 y x + 16 = 0
( y x − 4) 2 = 0
y x = 4.............(iii )
x
From (ii) we get, y = 2 2.............(iv)
x
From (iii) and (iv) we get, (2 2 ) x = 4
x2
2 2 = 22
x2
=2
2
x2 = 4
x = 2
2
From (iv) we get, y = 2 2
= 2 [when x = 2 ]
−2 1
y=2 2
= 2 −1 = [when x = −2 ]
2
Thus the solution is

33
x = -2
x = 2 
 or 1
y = 2 y= 
2
Exercise: Solve the following system of equations:
1 1

x3 + y3 = 6
x + y = 72
Solution: Given that,
1 1

x + y = 6.........................(i )
3 3

x + y = 72...........................(ii )
3
 13 1

From (i) we get,  x + y 3  = 6 3

 
1 1
 1 1

3 3 3
x + y + 3 x y  x + y 3  = 6 3
 
1

72 + 3( xy ) .6 = 6 33

12 + 3( xy ) 3 = 36
1

3( xy ) 3 = 24
1

( xy ) = 8
3

( xy ) = 8 3
xy = 512
512
y= .....................(iii )
x
Substituting the value of y [from (iii)] in (ii) we get,
512
x+ = 72
x
x 2 − 72 x + 512 = 0
x 2 − 64 x − 8 x + 512 = 0
x( x − 64) − 8( x − 64) = 0
( x − 64)( x − 8) = 0
Either x − 64 = 0 , or x − 8 = 0
 x = 64 or 8
When x = 64 then y = 8
Again, when x = 8 then y = 64
Thus the solution is
x = 64 x =8 
 or 
y =8  y = 64
Exercise: A commodity is produced by using 3 units of labour and 2 units of capital. The total
cost comes to 62. If the commodity is produced by using 4 units of labour and 1 unit of capital the
cost comes to 56. What is the cost per unit of labour and capital?
Solution: Let x = the cost per unit of labour
y = the cost per unit of capital.
34
According to the given problem, 3x + 2y = 62
4x + y = 56
By solving we get, x = 10 and y = 16
So the cost of per unit of labour is Tk.10 and the cost of per unit of capital is Tk.16.

Exercise: Tk.40,000 is invested in two shares, the first share yield 10% per annum and the second
3
share yield 12% per annum. If the total return at the end of one year is 10 %, how much was
4
invested at each rate.
Solution: Let Tk. x is invested at first share which gives return 10%. Then (40,000 – x) is invested
at second share that gives return 12%.
10 12(40,000 − x) 40,000  43
Therefore we can write, x+ =
100 100 100  4
x 3(40,000 − x)
+ = 4300
10 25
5 x + 240,000 − 6 x = 215000
x = 25000.
Hence Tk. 25,000 is invested at first share and 15,000 is invested at second share.

Exercise: A man’s annual income has increased for Tk. 4500 but there is no charge in the
income tax payable for him since the rate of income tax has been reduced from 7% to 5%. Find
the present income.
Solution: Let the previous annual income of the man = x.
Therefore, the present income of the man = x + 4500
7
Income tax for previous year = x 
100
5
Income tax for present year = ( x + 4500) 
100
According to the given condition,
7 5
x = ( x + 4500) 
100 100
7 x = 5 x + 22500
x = 11250
Hence the present income = x + 4500 = 11250 + 4500 = 15750.

Exercise: As the number of units manufactured increases from 4000 to 6000, the total cost of
production increases from Tk.22000 to 30000. Find the relationship between the cost (y) and the
number of units made (x) if the relationship is linear.
Solution: Let the relationship between x and y be given by
ax + by + c = 0
when x = 4000, y = 22000
4000a + 22000b + c = 0.................(i )
Again, when x = 6000, y = 30000
6000a + 30000b + c = 0.................(ii )
Multiplying (i) by 3 and (ii) by 2 we get
12000a + 66000b + 3c = 0................(iii )
12000a + 60000b + 2c = 0................(iv)
Subtracting (iv) from (iii) we get,
6000b + c = 0
35
−c
b=
6000
−c
Putting the value of b = in equation (i) we get,
6000
−c
4000a + 22000  +c =0
6000
c
a=
1500
c c
Thus the linear relation is x− y+c =0
1500 6000
4 x − y + 6000 = 0 ; which is the required equation.

Exercise: A pharmacist needs 100 gallons of 50% alcohol solution. She has on hand 20% alcohol
solution and 70% alcohol solution. How many gallons of each should she use to make the 100
gallons of 50% alcohol solution?
Solution: Let x = the number of gallons of 20% solution needed.
and y = the number of gallons of 70% solution needed.
Since a total of 100 gallons of final solution is needed we can write x + y = 100
Also since no alcohol is lost during the mixing process,
No. of gallons of alcohol in 20% solution + No. of gallons of alcohol in 70% solution =
No. of gallons of alcohol in 50% solution
In other words,
20% of x + 70% of y = 50% of 100
0.20 x + 0.70 y = (0.50)(100)
0.2 x + 0.7 y = 50
2 x + 7 y = 500
Combining this equation with the first equation produces the system
x + y = 100..........................(i )
2 x + 7 y = 500.......................(ii )
Multiplying equation (i) by 2 and (ii) by 1, we get,
2 x + 2 y = 200.............(iii )
2 x + 7 y = 500............(iv)
Subtracting (iv) from (iii) we get,
− 5 y = −300
y = 60
Now, putting the value of y = 60 in equation (i) we get, x = 40 .
Hence 40 gallons of 20% solution and 60 gallons of 70% solution are needed to make 100 gallons
of 50% solution.

Exercise: Demand for products of an industry is given by the equation pq = 500 ; where p is the
price and q is the quantity. Supply is given by the equation 30 + 2 p = q . Find the equilibrium
price and quantity.
Solution: The demand function is given by pq = 500 ............(i)
The supply function is 30 + 2 p = q ..........................(ii)
Substituting the value of q in (i) we get,
36
p (30 + 2 p ) = 500
30 p + 2 p 2 = 500
2 p 2 + 30 p − 500 = 0
p 2 + 15 p − 2500 = 0
p 2 + 25 p − 10 p − 2500 = 0
p ( p + 25) − 10( p + 25) = 0
( p + 25)( p − 10) = 0
Either ( p + 25) = 0 or ( p − 10) = 0 .
p = 10, - 25
Since price cannot be negative, so p = 10.
From (ii) we get q = 50.
Hence the equilibrium price and quantity are 10 and 50 respectively.

Exercise: If the law of demand and supply are respectively given by the equation 4q + 9 p = 48
q
and p = + 2 where p is the price and q is the quantity. Find the equilibrium price and quantity.
9
Solution: In the equilibrium point we know that demand = supply.
4q + 9 p = 48 ......................(i)
q
p = + 2 ...........................(ii)
9
q
Putting (ii) in (i) we get, 4q + 9( + 2) = 48
9
4q + q + 18 = 48
q=6
8
Again putting the value of q in (i) we get, p = .
3
8
Hence the equilibrium price and quantity are and 6 respectively.
3

Exercise: If the law of demand and supply are respectively given by the equation 2 p 2 + q 2 = 11
and p + 2q = 7 where p is the price and q is the quantity. Find the equilibrium price and quantity.
Solution: In the equilibrium point we know that demand = supply.
2 p 2 + q 2 = 11 ......................(i)
p + 2q = 7 ...........................(ii)
From (ii) we get, p = 7 − 2q .........................(iii)
Putting the value of (iii) in (i) we get,
2(7 − 2q) 2 + q 2 = 11
2(49 − 28q + 4q 2 ) + q 2 = 11
9q 2 − 56q + 87 = 0
9q 2 − 27q − 29q + 87 = 0
9q (q − 3) − 29(q − 3) = 0
(q − 3)(9q − 29) = 0
29
q = 3, .
9

37
5
Putting the value of q in (iii) we get, p = 1, .
9
5 29
Hence the equilibrium price and quantity are p = 1, and q = 3, respectively.
9 9

38
Matrix Algebra
Matrix: An Introduction

Learning Objectives:
After completing this lesson, you should be able to:
(i) Understand the nature of a matrix and matrix representation of data.
(ii) Define different types of matrices.

Introduction: J. J. Sylvester was the first to use the word ‘matrix’ in 1850 and later on in 1858
Arthur Cayley developed the theory of matrices in a systematic way. Matrix is a powerful tool of
modern mathematics and its study is becoming important day by day due to its wide applications
in every branch of knowledge. Matrix arithmetic is basic to many of the tools of managerial
decision analysis. It has an important role in modern techniques for quantitative analysis of
business and economic decisions. The tool has also become quite significant in the functional
business and economic areas of accounting, production, finance and marketing.

Matrix: Whenever one is dealing with data, there should be concern for organizing them in such
a way that they are meaningful and can be readily identified. Summarizing data in a tabular form
can serve this function. A matrix is a common device for summarizing and displaying numbers or
data. Thus, a matrix is a rectangular array of elements and has no numerical value. The elements
may be numbers, parameters or variables. The elements in horizontal lines are called rows, and
the elements in vertical lines are called columns.
A matrix is characterized further by its dimension. The dimension or order indicates the
number of rows and the number of columns contained within the matrix. If a matrix has m rows
and n columns, it is said to have dimension m  n , which is read m by n.
 a 11 a 12 a 13 
 
Example: A =  a 21 a 22 a 23 
 a a 33 
 31 a 32

Types of Matrices:

Row matrix: The matrix with only one row is called a row matrix or row vector.
Example: A = (2 3 4) .

Column matrix: The matrix with only one column is called a column matrix or column vector.
2 
 
Example: A =  3  .
4 
 

Note: Row matrix and column matrix are usually called as row vector and column vector respectively.
Square matrix: If the number of rows and the number of columns of a matrix are equal then the
matrix is of order n  n and is called a square matrix of order n.
1 2
Example: A =  .
3 4 

Rectangular matrix: If the number of rows and the number of columns of a matrix are not equal
then the matrix is called a rectangular matrix.
39
1 2 3
Example: A =  .
4 5 6 

Singular matrix: A square matrix A is said to be singular if the determinant formed by its
elements equal to zero or vanishes.
2 1
Example: Let A =   .
 4 2 
A = (2  2) − (4  1) = 0 . Hence A is a singular matrix.

Non-singular matrix:
A square matrix A is said to be non-singular if the determinant formed by its elements is non-zero.
5 3
Example: A =  
2 4 
A = (5  4) − (3  2) = 20 − 6 = 14 . Hence A is a non - singular matrix.

Null or Zero matrix: The matrix with all its elements equal to zero is called a null matrix or zero
matrix.
0 0
Example: A =  
0 0 

Diagonal matrix: A matrix whose all elements are zero except those in the principal diagonal is
called a diagonal matrix.

 a 11 0 0
 
Example: A =  0 a 22 0
 0 a 33 
 0

Scalar matrix: A diagonal matrix, whose diagonal elements are equal, is called a scalar matrix.
5 0
Example: A =  
0 5 

Sub-matrix: A matrix that is obtained from a given matrix by deleting any number of rows and
number of columns is called a sub-matrix of the given matrix.
5 3 2
1 2  
Example: A =   is a sub-matrix of B = 1 1 2
3 4 7 4 
 3

Unit matrix or Identity matrix: A matrix with every element in the principal diagonal equal to
one and all other elements equal to zero is called a unit matrix. A unit matrix is a square matrix.
It is denoted by I.
1 0
Example: A =  
0 1

Equal matrix: Two matrices A and B are said to be equal if their corresponding elements are
equal.
40
1 0 1 0
Example: Let A =   , B =   then A = B .
0 1 0 1

Transpose of a matrix: If the columns of a given matrix A are changed into rows or the rows are
changed into columns, the matrix thus formed is called the transpose of the matrix A and it is
generally denoted by AT.
1 2 3 1 4 7
   
Example: Let A =  4 5 6  then A =  2
T
5 8 .
7 9  3 9 
 8  6

Symmetric matrix: A square matrix A is called symmetric if it be same as its transpose so that A
= AT.
a h g a h g
   
Example: Let A =  h b f  then A T =  h b f
g c  g c 
 f  f
i.e. A = AT, so A is a symmetric matrix.

Skew-Symmetric matrix: A square matrix A is called skew-symmetric if AT = - A.

 0 h g 0 -h -g 
   
Example: Let A =  - h 0 f  then AT = A =  h
T
0 - f  = −A
- g 0  g 0 
 -f  f
i.e. A = − A , hence A is a skew-symmetric matrix.
T

Involuntary matrix: A square matrix A is called involuntary matrix provided it satisfies the
relation A2 = I, where I is the identity matrix.
1 1 
Example: A =  
 0 - 1
Idempotent matrix: A square matrix A is called idempotent matrix provided it satisfies the
relation A 2 = A .
 2 -2 4
 
Example: A =  - 1 3 4
 1 - 3 
 -2

Nilpotent matrix: A square matrix A is called nilpotent matrix of order m provided it satisfies the
relation A m = 0 and A m −1  0 , where m is a positive integer and O is the null matrix.
0 1
Example: A =   since A  0, A2 = 0
 0 0 

Complex conjugate of a matrix: It is a matrix obtained by replacing all its elements by their
respective complex conjugates.
2 + 3i 5  2 − 3i 5 
Example: If A =   then A =  
3 -3i 7  3 + 3 i 7 

41
Hermitian matrix: A matrix having complex elements of a square matrix A is a Hermitian matrix.
If ( A) = A , then A is called Hermitian matrix.

Skew-Hermitian matrix: A matrix having complex elements for matrix A.( A) = − A. A is skew
hermitian matrix.

Co-factor matrix
A matrix, which formed by the co-factor of the corresponding elements, is called co-factor matrix
and is denoted AC .
a11 a12 a13 
Example: If a matrix, A = a 21 a 22 a 23  then, the co-factor matrix
a31 a32 a33 

c11 c12 c13 


AC = c21 c22 c23 
 
c31 c32 c33 
Adjoint matrix:
The Adjoint matrix is the transpose of the cofactor matrix, that is adjA = A j = (cof A) T

Orthogonal matrix: A square matrix A is called an orthogonal matrix if AAT = AT A = I , where


I is an identity matrix and AT is the transpose matrix of A.
Questions for Review:
These questions are designed to help you assess how far you have understood and can apply the
learning you have accomplished by answering (in written form) the following questions:
1. What do you understand by matrix?
2. Why matrix algebra is so important in business and economics? Explain.
3. Discuss the various types of matrices.
4. In an examination, 20 students from college A, 30 students from college B and 40 students
from college C appeared. Only 15 students from each college could get through the
examination. Out of them 10 students from college A, 5 students from college B and 10
students from college C secured full marks. Write down the above data in matrix form.

Matrix Operations
Learning Objectives:
After completing this lesson, you should be able to:
(i) Express the concept of matrix operations.
(ii) Add, subtract and multiply of the given matrices.

Introduction: The operations of matrices are addition, subtraction, multiplication and division of
which addition and multiplication are the main operations. In this lesson we will discuss some of
the operations of matrix algebra.

42
Matrix Addition: Two matrices of the same dimensions are said to be conformable for addition.
The addition is performed by adding corresponding elements from the two matrices and entering
the result in the same row-column position of a new matrix.
If A and B are two matrices, each of size m  n then the sum of A and B is the m  n matrix
C whose elements are C ij = Aij + Bij ; i = 1, 2, 3… m and j = 1, 2, 3, ….n.

Properties of Matrix Addition:


(i) Commutative law: Matrix addition is commutative.
If A and B are two matrices of same order m  n , then A + B = B + A .
(ii) Associative law: Matrix addition is associative. If A, B and C are three matrices of same
order m  n , then A + ( B + C ) = ( A + B) + C .
(iii) Distributive law: If A and B are two matrices of same order m  n , and K is any scalar,
then K ( A + B) = KA + KB .
(iv) Existence of additive identity: If O denotes null matrix of the same order as that of A,
then A + O = A = O + A .
(v) Existence of an additive inverse: If A be any given m  n matrix and there exists
another m  n matrix B such that A + B = O = B + A ; where O be the m  n null
matrix.
(vi) Cancellation law: If A, B and C are three matrices of same order m  n , then
A+C = B+C  A = B.

Example 1: Find the sums A + B of the following matrices


8 9 13 4
A =   and B =  
12 7 2 6 
 8 + 13 9 + 4   21 13 
Solution: A + B =   =  
12 + 2 7 + 6  14 13 

Matrix Subtraction: The subtraction of two matrices is possible only when they are of the same
order. Such matrices are said to be conformable for subtraction. The subtraction is performed by
subtracting corresponding elements of the two matrices and entering the result in the same row-
column position of a new matrix.
If A and B are two matrices, each of size m  n then the sum of A and B is the m  n matrix
C whose elements are C ij = Aij − Bij ; i = 1, 2, 3… m and j = 1, 2, 3, ….n.

Example 2: Find the difference A − B of the following matrices


3 7 11 6 8 1
A =   and B =  
12 9 2 9 5 8 
3 − 6 7−8 11 - 1  − 3 -1 10 
Solution: A − B =   =  
12 - 9 9-5 2-8   3 4 - 6 

Scalar Multiplication of a Matrix: A matrix can be multiplied by a constant by multiplying each


component in the matrix by the constant. The result is a new matrix of the same dimensions as the
original matrix.
If K is any real number and A = [aij] is an m  n matrix, then the product KA is defined to
be the matrix whose components are given by K times the corresponding component of A i.e. KA
= [Kaij]

43
Laws of scalar multiplication:
(i) K ( A + B) = KA + KB
(ii) ( K1 + K 2 ) A = K1 A + K 2 A
(iii) IA = A
(iv) ( K 1 K 2 ) A = K 1 ( K 2 A) .
1 0 1
 
Example 3: If A =  2 1 2  . Find 5 A .
3 1 
 2
1 0 1  5 0 5 
   
Solution: 5 A = 5  2 1 2  = 10 5 10 
3 1  15 10 5 
 2
Multiplication of matrices: The number of columns of the first matrix must be equal to the
number of rows of the second matrix. Such matrices are said to be conformable for multiplication.
Let A be a matrix of order m  p and B be a matrix of order p  n . Then the product AB is defined
to be a matrix C of order m  n .

Properties of Matrix Multiplication:


(i) Associative law: Multiplication of matrices is associative i.e. A( BC ) = ( AB)C .
(ii) Distributive law: Multiplication of matrices is distributive with respect to matrix
addition i.e. A( B + C ) = AB + AC .
(iii) Multiplication of a matrix by a null matrix: If A is n  m and O is m  n matrices, then
AO = O = OA .
(iv) Multiplication of a matrix by a unit matrix: If A is a square matrix of order n  n and
I is the unit matrix of same order, then IA = A = AI .
(v) Multiplication of matrix by itself: If A is a square matrix then A. A = A 2 .

 2
 
Example 4: Find AB , where A = [9 11 3] and B =  6 
7
 
Solution: The matrices A and B are conformable for multiplication. The dimensions of AB are 1
 3 = 3  1 i.e. the product matrix AB will be 1  1 and a scalar, derived by multiplying each
element of the row vector by its corresponding element in the column vector and then summing
the products.

AB = [(9  2) + (11  6) + (3  7)] = 18 + 66 + 21 = 105.

2 3 1  1 2 -1 
Example 5: If A =   and B =  .
0 -1 5 0 -1 3 
Find (i) 3 A − 4 B .
(ii) 2 A − 3B .
2 3 1  1 2 -1
Solution: (i) 3 A − 4 B = 3   - 4  
0 -1 5 0 -1 3 
6 9 3  4 8 -4 
=   -  
0 - 3 15   0 - 4 12 

44
6 - 4 9-8 3 - (-4) 
=  
0 - 0 - 3 - (-4) 15 - 12 

2 1 7
=  
0 1 3 

2 3 1  1 2 -1
(iii) 2 A − 3B = 2   - 3  
0 -1 5 0 -1 3 
4 6 2  3 6 - 3
=   -  
0 - 2 10   0 - 3 9 
4 - 3 6-6 2 - (-3) 
=  
 0 - 0 - 2 - (-3) 10 - 9 
1 0 5
=  
0 1 1 

3 1 2 1 4
   
Example 6: If A =  0 1 1  and B =  2 2
1 0  1 0 
 2 
then find AB. Whether BA exists? Give reason.

3 1 2 1 4
   
Solution: AB =  0 1 1  2 2
1 0  1 0 
 2 
 3.1 + 1.2 + 2.1 3.4 + 1.2 + 2.0 
 
=  0.1 + 1.2 + 1.1 0.4 + 1.2 + 1.0 
1.1 + 2.2 + 0.1 1.4 + 2.2 + 0.0 

7 14 
 
= 3 2
5 8 

Here A is a matrix of order 3  3 and B is a matrix of order 3  2. Hence BA does not exist as number of
columns in B is not equal to the number of rows in A.

2 3 
 1 -2 3  
Example 7: If A =   and B =  4 5 .
- 4 2 5 2 
 1 
Find AB and show that AB  BA
2 3 
1 -2 3  
Solution: AB =     4 5 
- 4 2 5  
2 1 
 1.2 + (-2).4 + 3.2 1.3 + (-2).5 + 3.1
=  
 - 4.2 + 2.4 + 5.2 - 4.3 + 2.5 + 5.1 
45
 0 -4
=  
10 3 
2 3 
  1 -2 3
and BA =  4 5    
2 1  - 4 2 5 
 
 2.1 + 3.(-4) 2.(-2) + 3.2 2.3 + 3.5 
 
=  4.1 + 5.(-4) 4.(-2) + 5.2 4.3 + 5.5 
 2.1 + 1.(-4) 2.(-2) + 1.2 2.3 + 1.5 

 - 10 2 21 
 
=  - 16 2 37 
- 2 11 
 -2
Hence, AB BA.
1 2 2 1 0 0
   
Example 8: Evaluate A − 4 A − 5I , where A =  2
2
1 2  and I =  0 1 0
2 1 0 1
 2  0
1 2 2  1 2 2 9 8 8
     
2
Solution: A =  2 1 2  2 1 2 = 8 9 8
2 1  2 1 8 9 
 2 2  8

9 8 8 1 2 2 1 0 0
     
A − 4 A − 5I =  8
2
9 8 - 4 2 1 2 - 5 0 1 0
8 9  2 1 0 1
 8  2  0
9 - 4 + 5 8-8+ 0 8 - 8 + 0
 
= 8 - 8 + 0 9-4-5 8-8+ 0 
8 - 8 + 0 8-8+ 0 9 - 4 - 5 

0 0 0
 
= 0 0 0  = O, where O is a null matrix.
0 0 
 0

Questions for Review:


These questions are designed to help you assess how far you have understood and can apply the
learning you have accomplished by answering (in written form) the following questions:
 1 2
1. If A =   , find A2 + 3 A + 5I where I is unit matrix of order 2.
 − 3 0 
1 2 4 5
2. If A =   and B =   . Find a matrix C such that A + B = 2C .
3 4 6 7 
3 -3 4
 
3. If A =  2 -3 4  , find A 3 .
0 1 
 -1

46
1 2 
  3 5 9
4. Given A =  3 6  , B =  
5  6 - 2 1
 8 
(i) Write down the order of the matrices A and B.
(ii) Write down the order of the product AB.
(iii) Calculate AB.
(iv) Is it possible to calculate BA?
(v) Is AB = BA?
(vi) Are the following possible for operation?
A + B, A – B, 2B and A2

Determinant

Learning Objectives:
After completing this lesson, you should be able to:
(i) Understand the concept of determinant and advantages of determinant.
(ii) Express Cramer’s rule, and solve the system of linear equations by Cramer’s Rule.

Introduction:
The present lesson is devoted to a brief discussion of determinants and their more elementary
properties. The determinant concept is of particular interest in solving simultaneous equations.

Determinant: An important concept in matrix algebra is that of the determinant. If a matrix is


square, the elements of the matrix may be combined to compute a real- valued number called the
determinant and is denoted either by the symbol  , or by placing vertical lines around the
elements of the matrix or simple det. A. The signs of the successive terms in the expansion of
determinants will be alternately positive and negative until the last term is reached.

a11 a12 a13 


If, A = a 21 a 22 a 23 
a31 a32 a33 
a11 a12 a13
Determinant of A will be denoted by  = A = a 21 a 22 a 23
a31 a32 a33
Types of Determinants:

First Order Determinant: A determinant of first order is defined by the determinant of a 1  1


Matrix. The determinant of a 1  1 matrix is simply the value of the one element contained in the
matrix.
Let, A = [a11 ] be a square matrix. Then A = a11 be a determinant of first order.

Second Order Determinant: A determinant of second order is defined by the determinant of a 2


 2 Matrix.
a11 a12 
Let, A =   is a 2  2 matrix and the determinant of A is
a 21 a 22 

47
a11 a12
A= = a11a 22 − a12 a 21
a 21 a 22
That is the value of the determinant is given by the difference of the cross products.

Third Order Determinant: A determinant of third order is defined by the determinant of a 3  3


Matrix.
a11 a12 a13 
Let, A = a 21 a 22 a 23  is a 3  3 matrix and the determinant of A is
a31 a32 a33 

a11 a12 a13


A = a 21 a 22 a 23
a31 a32 a33

Minors and Cofactors: The method discussed earlier applies for calculating the determinant of
a 2  2 or 3 3 matrix. It does not, however, apply to matrices of higher dimensions. It is required
a procedure for calculating a determinant that applies to any square matrix. This procedure is
termed as the method of co-factor expansion. Before discussing the method of co-factor
expansion, we must define two terms minor and co-factor.

Minors:
The minor of an element defined as a determinant by omitting the row and the column containing
the element. Thus, a minor is the determinant of the sub matrix formed by deleting the i-th row
and j-th column of the matrix.
a11 a12 a13 
If a matrix, A = a 21 a 22 a 23 
a31 a32 a33 
a 22 a 23
then – minor of a11 = M 11 =
a32 a33
a 21 a 23
minor of a12 = M 12 =
a31 a33
a 21 a 22
minor of a13 = M 13 = and so on.
a31 a32
Co-factors:
The co-factor of an element is the co-efficient of the element in the expanded form and is equal to
the corresponding minor with proper sign. Thus, a cofactor is a minor with a prescribed sign. The
rules for the sign of a co-factor of any element = ( − 1) i + j  its minor, where i = number of row
and j = number of column.
The cofactor of aij = cij = (−1) i + j M ij
For example, cofactor of a11 = (−1)1+1 M 11 = M 11
cofactor of a12 = (−1)1+ 2 M 12 = − M 12

48
Example 1: Find the minor and co-factor of the element at the 1 st row of the determinant
1 2 3
4 5 0
3 2 7
5 0
Solution: The minor of the element 1 i.e. a11 is M 11 = = 35
2 7
4 0
The minor of the element 2 i.e. a12 is M 12 = = 28
3 7
4 5
The minor of the element 3 i.e. a13 is M 13 = = -7
3 2
The cofactor of 1 i.e. a11 is C11 = (−1)1+1 .35 = 35
The cofactor of 2 i.e. a12 is C12 = (−1)1+ 2 .28 = 28
The cofactor of 3 i.e. a13 is C13 = (−1)1+3 (-7) = -7

Expansion of Determinant and use of Sarrus Diagram:


a11 a12 a13
Let A = a 21 a 22 a 23
a31 a32 a33
If the co-factor of a11 , a12 and a13 are A11 , A12 and A13 respectively, then
A = a11 A11 + a12 A12 + a13 A13 .

Sarrus diagram: We can find out determinant value of a given matrix very conveniently by using
Sarrus diagram. It is found by the following process:
(i) Rewrite the first two columns of the matrix to the right of the original matrix.
(ii) Locate the elements on the three primary diagonals (P1, P2, P3) and those on the three
secondary diagonals (S1, S2, S3).
(iii) Multiply the elements on each primary and each secondary diagonal.
(iv) The determinant equals the sum of the products for the three primary diagonals
minus the sum of the products for the three secondary diagonals.

a11 a12 a13 


Let A = a 21 a 22 a 23  , the determinant may be found by the following process
a31 a32 a33 

S1 S2 S3
a11 a12 a13 a11 a12
a 21 a 22 a 23 a 21 a 22
a31 a32 a33 a31 a32

P1 P2 P3

49
Thus, algebraically the determinant value is computed as
A = (a11 a 22 a33 + a12 a 23 a31 + a13 a 21 a32 ) − (a31 a 22 a13 + a32 a 23 a11 + a33 a 21 a12 ) .
a11 a12 a13
Hence expansion of the determinant of a 21 a 22 a 23 will be
a31 a32 a33

A = a11 (a 22 a33 − a32 a 23 ) − a12 (a 21 a33 − a31 a 23 ) + a13 (a 21 a32 − a31 a 22 ) .


= a11 A11 + a12 A12 + a13 A13 .

1 5 3
Example 2: Find the value of 2 0 5
-4 1 -2

1 5 3
Solution: Let D = 2 0 5
-4 1 -2
= 1 (0 – 5) – 5 (- 4 + 20) + 3 (2 – 0)
= - 5 – 80 + 6 = 79.

Properties of Determinants:
Certain properties hold for determinants. The following properties can be useful in computing
the value of the determinant.
1) If two rows or columns are interchanged in a determinant the sign of the determinant
changes but its value is unchanged.
2) If rows are changed into columns and columns into rows, the determinant remains
unchanged.
3) If two rows or columns are identical in a determinant it vanishes.
4) If all the elements of any row or column are zero, the determinant is zero.
5) If any multiple of one row or column is added to another row or column, the value of the
determinant is unchanged.
6) If any row or column is a multiple of another row or column, the determinant equals to
zero.
1 1 1
Example 3: Show that a b c = ( a − b)(b − c )( c − a )
a2 b2 c2
Solution: Applying C1 = C1 – C2; C2 = C2 – C3 we get
0 0 1
a-b b-c c
a 2 - b2 b2 - c2 c2

50
1 1
= (a – b)(b – c)
a+b b+c
= (a – b)(b – c)(c – a)

a + b + 2c a b
Example 4: Show that c b + c + 2a b = 2( a + b + c ) 3
c a c + a + 2b
Solution: Applying C1 = C1 + C2 + C3, we get

2a + 2b + 2c a b
= 2a + 2b + 2c b + c + 2a b
2a + 2b + 2c a c + a + 2b
1 a b
= 2(a + b + c) 1 b + c + 2a b
1 a c + a + 2b

Applying R1 = R1 - R2, R2 = R2 - R23

0 - (a + b + c) 0
= 2(a + b + c) 0 (a + b + c) - (a + b + c)
1 a c + a + 2b
= 2(a + b + c)3.

1 x y+z
Example 5: Show that 1 y z+x =0
1 z x+y

Solution: Applying C 3 = C2 + C3 we get,


1 x x+y+z
= 1 y x+y+z
1 z x+y+z
1 x 1
= ( x + y + z) 1 y 1
1 z 1
= 0.
1 1 1
Example 6: Solve the equation x a b =0
3 3 3
x a b
Solution: Applying C2 = C2 – C1; C3 = C3 – C2 we get

51
1 0 0
x a-x b-a =0
x 3 3
a -x 3
b −a3 3

a-x b-a
=0
a3 - x3 b3 − a 3

( a − x )(b − a )(b 2 + ab + a 2 − a 2 − ax − x 2 ) = 0

( a − x )(b − a )(b 2 + ab − ax − x 2 ) = 0
− ( a − x )(b − a )( x 2 + ax − ab − b 2 ) = 0
( a − x )(b − a )( x 2 + ax − ab − b 2 ) = 0
− a  a 2 − 4( −ab − b 2 )
x = a or x =
2
− a  a 2 + 4ab + b 2 )
x = a or x =
2
x = a, b, − (a + b) .

Cramer’s Rule and Its Use in the Solution of Equations:

Cramer’s rule is a simple rule using determinants to express the solution of a system of linear
equations for which the number of equations is equal to the number of variables. This rule states
Di
xi = where xi is the i-th unknown variable in a series of equations, D is the determinant of
D
the coefficient matrix, and Di is the determinant of a special matrix formed from the original
coefficient matrix by replacing the column of coefficients of xi with the column vector of
constants. Thus, Cramer’s rule can be fruitfully applied in case D  0 .

Example 7: Solve the following system of equations by using Cramer’s Rule.


5 x − 6 y + 4 z = 15
7 x + 4 y − 3 z = 19
2 x + y + 6 z = 46
5 −6 4
Solution: Here D = 7 4 − 3 = 419
2 1 6
15 −6 4
Dx = 19 4 − 3 = 1257
46 1 6

52
5 15 4
Dy = 7 19 − 3 = 1676
2 46 6
5 −6 15
Dz = 7 4 19 = 2514
2 1 46
x y z 1
We know from the Cramer’s Rule, = = =
Dx D y Dz D
Dx 1257
Hence x = = =3
D 419
Dy 1676
y= = =4
D 419
D 2514
z= z = =6
D 419

Example 8: Solve the following system of equations by using Cramer’s Rule.


x+ y+z =9
2 x + 5 y + 7 z = 52
2x + y − z = 0

1 1 1
Solution: Here D = 2 5 7 = -4
2 1 -1
9 1 1
Dx = 52 5 7 = -4
0 1 -1
1 9 1
Dy = 2 52 7 = -12
2 0 -1
1 1 9
Dz = 2 5 52 = -20
2 1 0
x y z 1
We know from the Cramer’s Rule, = = =
Dx D y Dz D
Dx −4
Hence x = = =1
D −4
Dy − 12
y= = =3
D −4
D − 20
z= z = =5
D −4

53
Questions for Review:
These questions are designed to help you assess how far you have understood and can apply the
learning you have accomplished by answering (in written form) the following questions:

1. Find all the minors and co-factors of the following determinant


1 2 3
4 5 7
-2 8 1
1 a a2
2 Show that 1 b b 2 = (a − b)(b − c)(c − a )
1 c c2
a −b−c 2a 2a
3. Show that 2b b−c−a 2b = (a + b + c) 3
2c 2c c-a −b

x+ y x y
4. Find the value of x x+z z
y z y+z
5. Solve the following system of equations by using Cramer’s Rule:

x + 5y − z = 9
3x − 3 y + 2 z = 7
2 x − 4 y + 3z = 1
6. Solve the following system of equations by using Cramer’s Rule:
x − y + z =1
x + y − 2z = 0
2x − y − z = 0
p+x q+x r+x
7. Solve the equation q + x r+x p+x =0
r+x p+x q+x

Matrix Inversion

Learning Objectives:
After completing this lesson, you should be able to:
(i) Explain inverse matrix.
(ii) Solve system of linear equations by inverse matrix method.

Introduction: The operation of dividing one matrix directly by another does not exist in matrix
theory but equivalent of division of a unit matrix by any square matrix can be accomplished (in
most cases) by a process known as inversion of matrix. The concept of inverse matrix is useful in
solving simultaneous equations, input-output analysis and regression analysis.
54
Inverse Matrix: If A is a square matrix of order n, then a square matrix B of the same order n is
said to be inverse of A if AB = BA = I (unit matrix).
Methods of Matrix Inversion:
There are several methods for determining the inverse of a matrix; two of these are discussed in
below.
(i) Co-factor matrix method.
(ii) Gauss- Jordan Elimination method.

Working Rule for inverse matrix (Co-factor matrix method):


To evaluate the inverse of a square matrix A, we should follow the following steps:
(i) Evaluate A for the matrix A and be sure that A  0
(ii) Calculate the co-factors of all the elements of the matrix A.
(iii) Find the matrix of the co-factor AC.
(iv) Then find the Adjoint of A by taking transpose of AC such that Adj A = (AC)T.
(v) Finally divide all the elements of Adj A by A to get the required inverse A-1.

2 4
Example 1: Find the inverse of the matrix, A = 
3 8
2 4
Solution: The determinant of the matrix A is, A = =40
3 8
 8 -3 
The cofactor matrix of A is, A C =  
- 4 2 

 8 −4 
The Adjoint matrix of A is, A j = 
 -3 2 
Therefore, the inverse of A is,
1 1  8 −4 
A−1 = A j = 
 4  -3 2 
 1 2 0
Example 2: Find the inverse of the matrix, A =  1 0 − 1 
− 1 3 2 
1 2 0
Solution: The determinant of the matrix A is, A = 1 0 −1 = 1
−1 3 2
 3 −1 3
The cofactor matrix of A is, A = − 4
C
2 − 5
 
− 2 1 − 2
 3 −4 − 2

The Adjoint matrix of A is, A = − 1
J
2 1
 
 3 − 5 − 2 
Therefore, the inverse of A is,

55
 3 −4 − 2  3 −4 − 2
1 J 1
−1
A = A = −1 2 1 = − 1 2 1
 1  
 3 −5 − 2   3 −5 − 2 

Gauss-Jordan Elimination Method:


To determine the inverse of an m  m matrix ‘A’
(i) Determining the determinant value of A, whether it is non-singular or not.
(ii) Augment the matrix A with an m  m identity matrix, resulting in ( A I ).
(iii) Perform row operations on the entire augmented matrix so as to transform ‘A’
into an m  m identify matrix. The resulting matrix will have the form
( I A −1 ) where, the A −1 can be read to the right of the vertical line.
3 7 
Example 3: Find the inverse of the matrix, A =  
2 5
Solution: Augmented the matrix ‘A’ by 2  2 identity matrix, we get –
3 7 1 0 
 
2 5 0 1
 7 1 
 1 0 1
applying r1 = r1 
/
3 3
  3
2 5 0 1
 7 1 
1 3 3
0
  applying, r2 = r2 − r1  2
/

0 1

2
1
 3 3 

 7 1 
1 3 3
0
applying, r2 = r2 
/ 1
  3
0 1 −2 3

1 0 5 − 7 7
applying, r1 = r1 − r2 
/
0 1−2 
 3 3
 5 − 7
So, the inverse of ‘A’ is, A −1 = 
3
.
− 2

Solution of Linear Equations by Using Inverse of Matrix:


Matrix algebra permits the concise expression of a system of linear equations. The inverse matrix
can be used to solve a system of simultaneous equations. Let a system of linear equations are
a11 x + a12 y + a13 z = k1
a 21 x + a 22 y + a 23 z = k 2
a31 x + a32 y + a33 z = k 3

It can be written in the matrix form:


56
 a 11 a 12 a 13  x   k1 
    
 a 21 a 22 a 23  y  =  k 2 
a
 31 a 32 a 33  z   k 3 
 a 11 a 12 a 13  x  k1 
     
AX = B ; where, A =  a 21 a 22 a 23  , X =  y  , B =  k2 
a a 33  z  k 
 31 a 32    3
−1
X =A B

Example: Use matrix inversion to solve the following system of equations


4x 1 + x 2 − 5x 3 = 8
− 2x 1 + 3x 2 + x 3 = 12
3x 1 − x 2 + 4x 3 = 5
Solution: The given system of equations can be written in the matrix form

 4 1 − 5  x1  8 
− 2 3 1 x  = 12
   2  
 3 −1 4  x3  5 

X = A −1 B

4 1 -5
Now A = - 2 3 1 = 98
3 −1 4
13 11 -7 
The cofactor matrix of A is A C = 1 31 7 
16 6 14
13 1 16 
The Adjoint matrix of A is, A j = 11 31 6 
 
− 7 7 14
Therefore, the inverse of A is,
13 1 16 
1 
6 
−1 1
A = Aj = 11 31
 98  
− 7 7 14

13 1 16  8  2
1 
−1
X = A B= 11 31 6  12 = 5 
98      
− 7 7 14 5  1 

x1 = 2, x2 = 5, x3 = 1
Example: Solve the following system of equations by using Gaussian method.
57
x+ y+z =7
x + 2 y + 3 z = 16
x + 3 y + 4 z = 22
Solution: Given system of equations in matrix form
1 1 1  x  7 
1 2 3  y  = 16 

1 3 4  z  22

1 1 1  x  7 
0 1 2   y  = 9  ; Applying R = R – R
     2 2 1

0 2 3  z  15


R3 = R3 – R1

1 1 1x   7 
0 1  y  =  9 ; Applying R = R - 2R
2 
     3 3 2

0 0  z  − 3


- 1
x+ y+z =7
Hence y + 2z = 9
− z = −3
Thus, z = 3, y = 3, x = 1.

Example: Solve the following system of equations by using Gaussian method.


2x − 5 y + 7z = 6
x − 3y + 4z = 3
3 x − 8 y + 11z = 11
Solution: Given system of equations in matrix form
2 -5 7   x  6 
1 − 3 4   y  = 3 

3 - 8 11  z  11

1 -3 4   x  3 
2 −5 7   y  = 6 
    
3 -8 11  z  11

1 -3 4  x  3 
0 1 − 1  y  = 0 ; Applying R = R – 2R
     2 2 1

0 1 - 1  z  2


R3 = R3 – 3R1

58
1 -3 4  x   3
0 1 - 1   y  =  0  ; Applying R = R - R
     3 3 2

0 0 0   z   2

x − 3y + 4z = 3
Hence y−z =0
0=2
Since, 0 = 2 is false, the given system of equations has no solution. So given system of equations
is inconsistent.

Questions for Review:


These questions are designed to help you assess how far you have understood and can apply the
learning you have accomplished by answering (in written form) the following questions:
2 1 -1 

1. Find the inverse of the matrix, A =  0 2 1  .
 5 2 2 
 1 3 4 

2. Find the inverse of the matrix, A =  3 -1 6 
 - 1 5 1 
3. Solve the following system of equations by using Gaussian method.

2x − 5 y + 7z = 6
x − 3y + 4z = 3
3 x − 8 y + 11z = 11
4. Use matrix inversion to solve the following system of equations
x+ y+z =3
x + 2 y + 3z = 4
x + 4 y + 9z = 6
5. Use matrix inversion to solve the following system of equations
x + 2 y + 3z = 6
2x + 4 y + z = 7
3 x + 2 y + 9 z = 14

59
Application of Matrices in Business

Learning Objectives:
After completing this lesson, you should be able to:
(i) Develop matrices by using the given business information.
(ii) Apply the concepts of matrices to solve the business problems.

Introduction: Matrix is the powerful tool in modern mathematics having wide applications.
Demographers, sociologists, economists use matrices in different way. Many economic
relationships can be approximated by linear equations. Matrix algebra permits the concise
expression of a system of linear equations.

Illustrative Example:

Example 1: A manufacturer produces three products A, B, C that he sells in the market. Annual
sales volumes are indicated as follows:

Market Products
A B C
I 8000 10000 15000
II 10000 2000 20000

(i) If unit sale prices of A, B and C are $2.25, $1.50 and 1.25 respectively, find the total
revenue in each market with the help of matrices.
(ii) If the unit costs of the above three products are $1.60, $1.20 and $0.90 respectively, find
the gross profit with the help of matrices.

Solution: (i) The total revenue in each market is given by the product matrix
 8000 10000 
 
(2.25 1.50 1.25) 10000 2000 
15000 20000 

= [51750 50500]
The total revenue from the market I is $51750 and the total revenue from the market II is $50500.
(ii) The total cost of products with the manufacturer sells in the markets are

 8000 10000 
 
(1.60 1.20 0.90) 10000 2000 
15000 20000 

= [38300 36400].
The total cost of products that the manufacturer sells in the market I and II are $38300 and $36400
respectively.
Required gross profit = (Total revenue received from both the markets) – (Total cost of products
that the manufacturer sold in both the market).
= (51750+50500)- (38300+36400)
= 102250 – 74700
= 27550.

60
Example 2: A finance Company has offices located in every division, every district and every
Thana. Assume that there are five divisions, thirty districts and two hundred Thanas. Each office
has one head clerk, one cashier, one clerk and one peon. A divisional office has, in addition one
office superintendent, two clerks, one typist and one peon. A district office has in addition one
clerk and one peon. The basic monthly salaries are as follows: office superintendent $500, head
clerk $200, cashier $175 clerks and typists $150 and peon $100. Using matrix notation, find the
following
(i) The total number of posts of each kind in all the offices taken together.
(ii) The total basic monthly salary bill of each kind of office and
(iii) The total basic monthly salary bill of all the offices taken together.

Solution: Let the number of offices can be arranged as elements of a row matrix
A = (5 30 200)
The composition of staff in various offices can be arranged in a 3  6 matrix
1 1 1 3 1 2
 
B = 0 1 1 2 0 2
0 1 
 1 1 1 0

The basic monthly salaries of various types of employees of these offices correspond to the
 500 
 
 200 
175 
elements of the column matrix, C =  
150 
150 
 
100 
 
(i) Total number of posts of each kind in all the offices are the elements of the product
matrix AB.

1 1 1 3 1 2
 
(5 30 200)  0 1 1 2 0 2
0 1 
 1 1 1 0
= [5 235 235 275 5 270]

Thus, the required numbers of posts in all the offices taken together are 5-office superintendent,
235 head clerks, 235 cashiers, 275 clerks, 5 typists and 270 peons.

 500 
 
 200 
1 1 1 3 1 2  1675 
  175   
(ii)  0 1 1 2 0 2   =  875 
0 150  
 1 1 1 0 1     625 
150
 
100 
 
Thus, the total basic monthly salary bill of each divisional, district and Thana offices are $1675,
$875 and $625 respectively.

61
(iii) Total basic monthly salary bill of all the offices is the element of the product matrix ABC i.e.
1675 
 
(5 30 200)   875  = 159625.
 625 
 
Thus, the total basic monthly salary bill of all the offices taken together is $159625.

Example 3: Three persons A, B and C posses Tk.3000, Tk.2000 and Tk.2500 respectively. A with
his entire amount purchased 5 shares of Tk. X each, 3 shares of Tk. Y each and 4 shares of Tk. Z
each. B purchased 3 shares of Tk. X each, 4 shares of Tk. Y each and 2 shares of Tk. Z each with
his entire amount and C purchased 4 shares of Tk. X each, 3 shares of Tk. Y each and 4 shares of
Tk. Z each with his entire amount. Determine the value of each share of different types.

Solution: We have, 5x + 3y + 4z = 3000


3x + 4y + 2z = 2000
4x + 3y + 4z = 2500
5 3 4
D= 3 4 2 = 10
4 3 4
3000 3 4
Dx = 2000 4 2 = 5000
2500 3 4
5 3000 4
Dy = 3 2000 2 = 1000
4 2500 4
5 3 3000
Dz = 3 4 2000 = 500
4 3 2500
From Cramer’s rule we know that,
x y z 1
We know from the Cramer’s Rule, = = =
Dx D y Dz D
Dx 5000
Hence x = = = 500
D 10
Dy 1000
y= = = 100
D 10
D 500
z= z = = 50
D 10

Example 4: To control a certain crop disease it is necessary to use 7 units of chemical A, 10 units
of chemical B and 6 units of chemical C. One barrel of spray P contains 1 unit of A, 4 units of B
62
and 2 units of C. One barrel of spray Q contains 3 units of A, 2 units of B, and 2 units of C. One
barrel of spray R contains 4 units of A, 3 units of B and 2 units of C. How much of each type of
spray should be used to control the disease?

Solution: Let x barrels of spray P, y barrels of spray Q and z barrels of spray R be used to control
the disease. Then we can write,
x + 3y + 4z = 7
4 x + 2 y + 3 z = 10
2x + 2 y + 2z = 6
The given information can be written under the matrix form as follows:
 1 3 4   x  7 
4 2 3   y  = 10

 2 2 2   z  6 
−1
x   1 3 4  7 
 y =  4 2 3  10
    
 z   2 2 2  6 
 1 3 4 

Let A =  4 2 3 
 2 2 2 
1 3 4
The determinant of the matrix A is, A = 4 2 3 =8
2 2 2
 -2 -2 4 
The cofactor matrix of A is, A =  2 C
-6 4 
 1 13 − 10
 -2 2 1 
The Adjoint matrix of A is, A = − 2
J
-6 13 
 4 4 − 10 
Therefore, the inverse of A is,
 -2 2 1 
1 J 1 
−1
A = A = − 2 -6 13 
 8
 4 4 − 10 

x   -2 2 1  7 
 y  = 1 − 2 -6 13  10 .
  8  
 z   4 4 − 10  6 

x  12
 y  = 1 4 
  8 
 z  8 

63
3 
x   2
 y  = 1 2  .
   
 z  1 
 
1 1
Hence 1 barrels of the spray P, barrel of spray Q and 1 barrel of spray R should be used to
2 2
control the disease.

Example 5: The cost of manufacturing the three types of motorcars is given below:
Car Labour hours Material used Subcontracted works
A 40 100 50
B 80 150 80
C 100 250 100
Labour cost $2 per hour, per unit material cost is $0.5 and one unit of subcontracted work cost s
$1. Find the total cost of manufacturing 3000, 2000 and 1000 vehicles of type A, B, C respectively.
If the selling price of car A, B, C is $2000, 3500 and $4500 respectively, then find the profit from
selling those cars.

Solution: Consider the following matrices,


40 100 50  2 
M = 80 150 80  N = 10
   
100 250 100 1 
180 
MN = 315
 
425
This column matrix represents cost of each car A, B, C in that order.
Let P = (3000 2000 1000), this row matrix represents number of cars A, B, C to be manufactured
in that order.
Now PMN = (1595000)
Thus total cost of manufacturing three cars A, B, C is $1595000.
2000
Let Q = 3500 ; this column matrix represents the selling price of A, B, C.
 
4500
2000
Now, Total Revenue = PQ = (3000 2000 1000) 3500
 
4500
= 17500000
Profit = Total revenue – Total cost = 17500000 – 1595000 = 15905000.

Questions for Review:


These questions are designed to help you assess how far you have understood and can apply the
learning you have accomplished by answering (in written form) the following questions:

64
1. A, B and C has Tk. 480, Tk. 760 and Tk.710 respectively. They utilized the amounts to
purchase three types of shares of prices x, y and z respectively. A purchases 2 share of
price x, 5 of price y and 3 of price z. B purchases 4 shares of price x, 3 of price y and 6 of
price z. C purchases 1 share of price x, 4 of price y and 10 of price z. Find the value of x,
y and z.
2. A manufacturing unit produces three types of television sets A, B, C. The following matrix
shows the sale of television sets in two different cities.
A B C 
 
 400 300 200 
 300 100 
 200
If cost price of each set A, B, C is Tk.1000, Tk.2000, and Tk.3000 respectively and selling
price Tk.1500, Tk.3000, Tk.4000 respectively, find the total profit using matrix algebra only.
3. The following matrix represents the results of the examination of MBA.
1 2 3 4
 
5 6 7 8
9 12 
 10 11
The rows represent the three sections of the class. The first three columns represent the
number of students securing 1st, 2nd and 3rd divisions respectively in that order and fourth
column represents the number of students who failed in the examination.
(i) How many students passed in three sections respectively?
(ii) How many students failed in three sections respectively?
(iii) Write down the matrix in which number of successful students is shown.
(iv) Write down the column matrix where only failed students are shown.
(v) Write down the column matrix showing students in 1st division from three
sections.
4. A publishing house has two branches. In each branch, there are three offices. In each office, there are 3
peons, 4 clerks and 5 typists. In one office of a branch, 6 salesmen are also working. In each office of
other branch 2 head clerks are also working. Using matrix notation find
(i) the total number of posts of each kind in all the offices taken together in each
branch.
(ii) the total number of posts of each kind in all the offices taken together from both
the branches.

65

You might also like